Sie sind auf Seite 1von 101

Questions

Specialty: MRCP 1 May 2014


Question: 0
A 42-year-old woman who is undergoing chemotherapy for carcinoma of the breast presents to the Emergency Department some 10 days post completion of her last
cycle of treatment. She has suffered from symptoms of pharyngitis with a fever that has not responded to paracetamol over the last 24hrs. Examination reveals a fever of
38.5C. Her tonsils are enlarged and erythematous. Investigations;
Hb 9.1 g/dl
WCC 3.1 x10
9
/l
Neutrophils 0.2 x10
9
/l
PLT 95 x10
9
/l
Na
+
137 mmol/l
K
+
4.3 mmol/l
Creatinine 105 micromol/l
CRP 135 mg/l
Which of the following is the most appropriate antibiotic regimen?
A Ciprofloxacin and Vancomycin YOUR ANSWER
B Cefuroxime and Gentamicin
C Gentamicin and Teicoplanin
D Piperacillin and Tazobactam CORRECT ANSWER
E Vancomycin and Gentamicin
The answer is Piperacillin and Tazobactam - NICE guidelines are very clear with respect to initial therapy of neutropaenic sepsis:Offer beta lactam
monotherapy with piperacillin with tazobactam as initial empiric antibiotic therapy to patients with suspected neutropenic sepsis who need intravenous treatment
unless there are patient-specific or local microbiological contraindications.Do not offer an aminoglycoside, either as monotherapy or in dual therapy, for the
initial empiric treatment of suspected neutropenic sepsis unless there are patient-specific or local microbiological indications.The guidelines also recommend
that within 24hrs of presentation to secondary or tertiary care, patients should be managed by an oncology specialist (i.e. handed over to the specialist
oncology firm at the latest the day after admission). http://www.nice.org.uk/nicemedia/live/13905/60866/60866.pdf
Specialty: MRCP 1 May 2014
Question: 1
A 16-year-old male comes to the Endocrinology Clinic for review. He is as tall as his peers but his parents are concerned because he does not appear to have entered
puberty. Lack of a sense of smell has been noted, although this was thought to have at least in part been related to a rugby injury a few years earlier. Examination reveals
normal height (1.9m). His BP is 105/80 mmHg; pulse is 66/min and regular. He has small testes (volume around 4ml). There is decreased facial and body hair. Which of
the following would you consider as the most likely diagnosis?
A Congenital adrenal hyperplasia YOUR ANSWER
B Constitutional delayed puberty
C Kallmanns syndrome CORRECT ANSWER
D Kleinfelters syndrome
E Testicular feminisation
The answer is Kallmanns syndrome - The lack of sense of smell and the fact that this child is of normal height steers us away from constitutional delayed
puberty and towards Kallmanns syndrome. If fertility is required then GnRH or gonadotrophin replacement can restore it. If fertility is not required, then
sex steroid replacement with testosterone is a reasonable option. Congenital adrenal hyperplasia leads to virilisation; testicular feminisation is associated
with female phenotype. Kleinfelters would be a possible alternative diagnosis, but is not associated with a lack of sense of smell.
Specialty: MRCP 1 May 2014
Question: 2
A 70-year-old man presents to the Rheumatology Clinic with increasingly severe back pain, which began as lumbar spine pain, spread to involve both buttocks, and now
affects the tops of his legs. The pain is worst on walking, and he actually feels like his legs are going to give way unless he sits town to rest when it gets really bad. It is
relieved when he sits, particularly when he leans forward as well. He is able to cycle without pain. There is a history of smoking 10 cigarettes per day and he has mild
hypertension. There is some loss of the lumbar lordosis, although neurological examination of both lower limbs including power, tone and reflexes appears
unremarkable. All peripheral pulses are palpable in both lower limbs. Which of the following is the most likely diagnosis?
A Ankylosing spondylitis
B Intermittent claudication
C Lumbar disc prolapse
D Lumbar discitis
E Lumbar spinal stenosis CORRECT ANSWER
The answer is Lumbar spinal stenosis - The picture of pain on exercise, relieved by sitting forward and resting is typical of lumbar spinal stenosis. A major
difference versus vascular claudication is that sitting aerobic exercise, (e.g. cycling) can take place without any pain at all. Lumbar disc prolapse is usually
associated with pain affecting one leg worse than the other, and neurological signs in the affected nerve root distribution, e.g. L4/5 or L5/S1. Lumbar spine x-
ray and MRI are mainly indicated for ruling out other potential conditions. Evidence for surgical intervention is only present in those patients who have
degenerative lumbar spine disease, which has precipitated the stenosis symptoms.
Specialty: MRCP 1 May 2014
Question: 3
A 45-year-old man comes to the Neurology Clinic for review. He is very concerned that his 78-year-old father has recently been diagnosed with Alzheimers disease,
and requests information on assessment of his possible risk factors for developing the disease. He has mild hypertension controlled with Ramipril 5mg daily and holds
down a stressful job as an accountant. He takes NSAIDs after exercise because of a previous knee injury. Examination reveals a BP of 138/82 mmHg; pulse is 74/min
and regular. His BMI is 28.Which of the following would be considered the most important risk factor for developing Alzheimers in this case?
A NSAID use
B BMI 28
C Controlled hypertension
D Family history of Alzheimers CORRECT ANSWER
E Stressful job as an accountant
The answer is Family history of Alzheimers - A number of risk factors have been identified for Alzheimers including hypertension and obesity, although in
this case with only a mildly elevated BMI and controlled BP, family history provides a much greater contribution to overall risk. A complex job, such as
working as an accountant may actually be protective against development of the disease, as may use of NSAIDs.Identified risk factors for Alzheimers are
listed below:
Advancing age
Family history
APOE 4 genotype
Obesity
Insulin resistance
Vascular factors
Dyslipidemia
Hypertension
Inflammatory markers
Downs syndrome
Traumatic brain injury
Specialty: MRCP 1 May 2014
Question: 4
A 28-year-old nursery nurse is admitted with a headache, neck stiffness and rash, and found to have pneumococcal meningitis. She is treated with high dose IV
antibiotics. No other children or staff members at the nursery school have reported any symptoms of infection at all. Which of the following is the most appropriate
intervention with respect to prophylaxis for other staff and students?
A Do nothing CORRECT ANSWER
B Ceftriaxone
C Ciprofloxacin
D Penicillin V
E Rifampicin
The answer is Do nothing - Whilst Ciprofloxacin is the first line prophylaxis for N. meningitidis meningitis, no prophylaxis is required for pneumococcal
meningitis. In patients requiring meningococcal prophylaxis for which Ciprofloxacin is unsuitable, Rifampicin is the most appropriate intervention.
Ceftriaxone, Rifampicin and Azithromycin are other potential choices. http://www.hpa.org.uk/webc/hpawebfile/hpaweb_c/1194947389261
Specialty: MRCP 1 May 2014
Question: 5
A 30-year-old woman who is 30 weeks pregnant with her first child comes to the clinic for review. She has a history of asthma and stepped up her therapy to
Beclomethasone 200mcg total daily dose and Salmeterol 50mg BD some 4 weeks earlier. She is still appreciably short of breath, and waking 2-3 times per night to use
her Salbutamol inhaler. Examination reveals a BP of 105/70, pulse is 82 and regular. There is scattered wheeze and good bilateral air entry on auscultation of the chest.
Her PEFR is 310 l/min (550 predicted).Which of the following is the most appropriate next step?
A Add oral steroids
B Add Monteleukast
C Add Theophylline
D Add Ipratropium bromide
E Increase inhaled steroid CORRECT ANSWER
The answer is Increase inhaled steroid - In this situation the inhaled steroid could be increased to Beclomethasone 800mcg total daily dose or equivalent (for
example, she could be switched to a combination corticosteroid, long acting beta agonist inhaled product). If she still had symptoms despite maximising inhaled
corticosteroids, addition of Monteleukast would be the most appropriate next option. Cumulative safety data suggests a benign risk profile for Monteleukast in
the pregnancy setting. Theophylline is not recommended in this setting, and oral steroids could contribute to significant weight gain, as such they are avoided if
possible.
Specialty: MRCP 1 May 2014
Question: 6
A 62-year-old man who suffered a TIA some 6 years ago comes to the clinic for review. He is taking a combination of Aspirin and Dipyridamole. Which of the
following correctly describes the mechanism of action of Dipyridamole?
A ADP receptor inhibitor
B Phosphodiesterase inhibitor CORRECT ANSWER
C COX-1 inhibitor
D Factor Xa inhibitor
E Thrombin inhibitor
The answer is Phosphodiesterase (PDE) inhibitor - Dipyridamole is a PDE inhibitor that leads to an increase in intracellular cyclic AMP, and this in turn
blocks the platelet response to ADP. Its use in combination with Aspirin for stroke prevention has fallen out of favour in recent years, as Clopidogrel has been
recommended as an alternative by both the RCP and NICE. Clopidogrel is an ADP receptor inhibitor. NSAIDs including aspirin are inhibitors of COX-1 and
COX-2. Direct factor Xa inhibitors have recently become available for prophylaxis against VTE and embolism in AF. A useful advantage of these agents is
that they do not require INR monitoring. Oral direct thrombin inhibitors have a role in the treatment of ACS, a role as an alternative to heparin when
heparin-induced thrombocytopaenia (HIT) occurs, and utility in venous thrombolism prophylaxis.
Specialty: MRCP 1 May 2014
Question: 7
A 56-year-old woman with a 35-year history of Type 1 diabetes comes to the clinic for review. Her main complaint is of worsening pain and stiffness of her right thumb,
coupled with thickening of the skin over the palmar surface. On examination there is weakness of thumb apposition on the right hand side. Examination of the left hand
reveals normal power and there is no sensory loss. Investigations;
Hb 13.0 g/dl
WCC 7.1 x10
9
/l
PLT 197 x10
9
/l
Na
+
138 mmol/l
K
+
4.1 mmol/l
Creatinine 122 micromol/l
Glucose 7.1 mmol/l
ESR 13 mm/1
st
hour
CRP 15 mg/l
X-ray both hands unremarkable
Which of the following is the most likely diagnosis?
A Cheiroarthropathy CORRECT ANSWER
B Rheumatoid arthritis
C Osteoarthritis
D Psoriatic arthritis
E Gout
The answer is Cheiroarthropathy - Diabetic cheiroarthropathy is associated with prolonged duration of diabetes and the presence of microvascular
complications. A collection of conditions including frozen shoulder and carpal tunnel is described as making up cheiroarthropathy, and thickened swollen skin
over the fingers is recognised as one manifestation of the disorder. Improved glucose control is advocated in an attempt to reduce further deterioration in
musculoskeletal function, coupled with physiotherapy. The near normal ESR and CRP count against a diagnosis of inflammatory arthritis.
Specialty: MRCP 1 May 2014
Question: 8
A 23-year-old man with no known psychiatric history becomes acutely disturbed in the Emergency Department and is behaving in a way that is putting him and others at
risk. You and the staff have attempted to talk him down and have offered support and reassurance. You have also alerted the security team and have mobilised enough
staff to make the situation safer. You now have to proceed to drug treatment. He is refusing oral medication. What drug would you give?
A Diazepam im
B Lorazepam im
C Chlorpromazine im
D Haloperidol im CORRECT ANSWER
E Quetiapine im
Acutely disturbed behaviour
Acutely disturbed behaviour can be a feature of any medical disorder
Urgent, necessary treatment can be given, without consent, under Common Law
Rapid tranquillisation should only occur when adequate staff for supervision and observation are available
A tranquillised patient should then be carefully monitored, and the causes of the acute disturbance investigated
Antipsychotic medication such as Haloperidol is the first line medication recommended by NICE guidelines unless there is a history of Parkinson's or other
disorder which may be exacerbated by dopamine antagonists
Where there is Parkinson's, Benzodiazepines would therefore be the first-line drug of choice
Lorazepam is preferable to Diazepam as it is faster acting
also, Diazepam is not well absorbed via the intramuscular route
Intramuscular Chlorpromazine should be avoided as it can be painful when delivered by this route
Quetiapine is only available in an oral formulation
http://www.nice.org.uk/nicemedia/live/13060/49909/49909.pdf
Specialty: MRCP 1 May 2014
Question: 9
A 43-year-old man who is known to have HIV and is poorly compliant with his anti-retroviral medications comes to the Emergency Department with shortness of
breath, particularly when he undertakes mild physical exercise. He has also had a dry cough which has increased significantly over the past few days. Examination
reveals a pyrexia of 37.9C, his BP is 122/71 mmHg, pulse is 85/min and regular. He has scattered wheeze and coarse crackles on auscultation of the chest. He
desaturates by 3% on pulse oximeter when you ask him to walk twice up and down the corridor.You suspect pneumocystis jirovecii (PCJ) pneumoniaWhich of the
following would be most suggestive of the condition?
A Elevated 1,3 beta-D glucan levels CORRECT ANSWER
B Elevated alkaline phosphatase
C Elevated LDH
D Ground glass infiltration on CT thorax
E Patchy consolidation on CXR
The answer is Elevated 1,3 beta-D glucan levels (BDG) - Beta-D glucan (BDG) is a polysaccharide present in the walls of a number of invasive fungi and in the
walls of pneumocystis jirovecii cysts. As such testing for elevated BDG levels is currently under development as a screening option for PCJ pneumonia.
Elevated LDH, patchy consolidation on CXR and ground glass infiltration on CT thorax are all found in association with PCJ pneumonia, but none of these
features is specific for the condition. In severe PCJ pneumonia IV Co-trimoxazole is the drug of choice; in those who cannot tolerate it, Pentamidine is an
alternative option. http://www.ncbi.nlm.nih.gov/pmc/articles/PMC3256688/
Specialty: MRCP 1 May 2014
Question: 10
You are asked to see a 63-year-old man who was admitted to the Coronary Care Unit a few hours ago after PCI for an inferior myocardial infarction. The nurses are
concerned because he is hypotensive and relatively bradycardic with a BP of 90/60 mmHg and a pulse of 65/min. His JVP is elevated. O2 saturation is 93% and his chest
is clear on auscultation. You decide to insert a jugular line to aid with respect to his fluid management. Quite soon after the procedure he deteriorates with increased
shortness of breath and decreased right sided breath sounds on auscultation. His saturation drops to 89% and a portable chest film indicates right lower lobe collapse.
Which of the following is the most likely complication of line insertion which has occurred?
A Air embolism
B Chylothorax
C Haemothorax
D Pneumothorax CORRECT ANSWER
E Venous thromboembolism
The answer is Pneumothorax - Pneumothorax is seen in up to 15% of patients who undergo subclavian line insertion, although it is much more rarely seen in
conjunction with jugular venous puncture. The lung collapse and drop in saturation raises the possibility of pneumothorax. In this situation chest drain
insertion is required. Chlyothorax takes some time to accumulate and would be suggested by a pleural effusion appearing over a few days. Haemothorax would
be suggested by a reduction in BP, and local pressure effects from haematoma formation may be seen. Air embolism would not lead to partial lung collapse but
would lead to shortness of breath and cyanosis. Venous thromboembolism is seen where clotting occurs around the venous catheter tip, although this would not
be seen so soon after the procedure.
Specialty: MRCP 1 May 2014
Question: 11
A 29-year-old man comes to the Emergency Department complaining of pain on passing urine, a dry cough and arthralgia, symptoms of which have increased over the
past few days. He admits to unprotected sex with two new partners during a holiday to Spain from which he has recently returned. He has no past medical history apart
from mild asthma, which is well controlled with a PRN Salbutamol inhaler. On examination his BP is 121/81 mmHg; pulse is 75/min and regular. He has scattered
wheeze on auscultation of the chest. He has a temperature of 37.9C. Investigations;
Hb 10.2 g/dl (red cell agglutination seen on blood film)
WCC 9.0 x10
9
/l
PLT 171 x10
9
/l
Na
+
137 mmol/l
K
+
4.3 mmol/l
Creatinine 100 micromol/l
Urine pus cells seen but no organisms identified
Which of the following is the most likely diagnosis?
A Chlamydia trachomatis
B Mycoplasma hominis CORRECT ANSWER
C Neisseria gonorrhoeae
D Trichomonas vaginalis
E Treponema pallidum
The answer is Mycoplasma hominis - A sexually transmitted infection associated with a dry cough, arthritis and haemaglutination on the blood film raises the
possibility of M. hominis infection. Given that the organism is intracellular and requires special media for growth, it is not surprising that initial microscopy is
negative. Oral tetracyclines were formerly the treatment of choice, although resistance rates of up to 30-40% have been reported. Macrolides, fluoroquinolones
and Clindamycin are all potential alternative agents. Macrolides do of course also cover for both gonorrhoea and chlamydia infection and may therefore
represent the optimal therapeutic approach here.
Specialty: MRCP 1 May 2014
Question: 12
A 43-year-old woman with a long-term history of manic depressive psychosis comes to the Emergency Department. She has nausea, diarrhoea and light-headedness and
feels weak and lethargic. Her Lithium level is measured at 2.5 mmol/l. Recent history of note is that she has begun a course of Metronidazole for a dental abscess.Which
of the following is the most likely cause of her Lithium toxicity?
A Decreased GI excretion
B Decreased protein binding
C Increased absorption
D Reduced renal clearance CORRECT ANSWER
E Reduced metabolism
The answer is Reduced renal clearance - Metronidazole is thought to reduce renal clearance of Lithium and this is likely to have driven the elevated Lithium
level and symptoms of toxicity seen here. Lithium is not protein bound, therefore Metronidazole does not have any additional / substantive effect on protein
binding in this situation. Lithium is not metabolised in the liver and 95% or more of any dose is excreted in the urine. Lithium is well absorbed from the GI
tract.Other agents known to increase Lithium levels are listed below:
Non-steroidal anti-inflammatory drugs, including cyclo-oxygenase (COX) 2 inhibitors (monitor serum lithium concentrations more frequently if NSAID
therapy is initiated or discontinued)
Angiotensin-converting enzyme (ACE) inhibitors
Angiotensin II receptor antagonists
Diuretics (thiazides show a paradoxical antidiuretic effect resulting in possible water retention and Lithium intoxication). If a thiazide diuretic has to be
prescribed for a Lithium-treated patient, Lithium dosage should first be reduced and the patient re-stabilised with frequent monitoring. Similar
precautions should be exercised on diuretic withdrawal. Loop diuretics seem less likely to increase lithium levels
Other drugs affecting electrolyte balance, e.g. steroids, may alter Lithium excretion and should therefore be avoided
Tetracyclines
http://www.medicines.org.uk/emc/medicine/6981/SPC/Priadel+Liquid/#PHARMACOKINETIC_PROPShttp://www.medicines.org.uk/emc/medicine/24125/SPC/Metronidazole+Tablets+500+mg/#PHARMACOKINETIC_PROPS
Specialty: MRCP 1 May 2014
Question: 13
A 42-year-old woman comes to the Rheumatology Clinic for review. She has multiple small joint pains, and is suffering from nocturnal fevers and anorexia. Her anti-
nuclear antibody result is positive.Which is the most likely immunoglobulin class for the ANA seen here?
A IgA
B IgD
C IgE
D IgG CORRECT ANSWER
E IgM
The answer is IgG - Whilst IgM and IgA anti-nuclear antibodies are found, by far the majority are IgG antibodies. The presence of antibodies is usually
initially detected using ELISA, and a titre is then calculated using immunofluorescence. Any titre >1 in 160 is considered a clinically significant positive result.
The frequency of positive ANA testing on Hep-2 cells via immunofluorescence is detailed below:
Mixed connective tissue disease: 100%
Drug-induced lupus erythematosus: 100%
Systemic lupus erythematosus: 95%-100%
Sjgren syndrome: 80%
Scleroderma: 60%-95%
Polymyositis-dermatomyositis: 49%-74%
Rheumatoid arthritis: 40%-60%
Normal: Less than 4%
Specialty: MRCP 1 May 2014
Question: 14
A 67-year-old smoker recently underwent radiotherapy for an inoperable lung carcinoma. On examination is cachectic and jaundiced. Laboratory results reveal a serum
sodium concentration of 120 mmol/l. Which one of the following is the most likely cause?
A Sodium-restricted diet
B Sodium-reduced water drinking
C Syndrome of inappropriate antidiuretic hormone secretion CORRECT ANSWER
D Liver metastases
E Bone metastases
Syndrome of inappropriate antidiuretic hormone secretion in cancer patients- Aetiopathogenesis and presentation
The continued secretion of antidiuretic hormone (ADH) in an amount in excess of the bodys needs leads to overhydration in both the intracellular and
extracellular compartments, the so-called syndrome of inappropriate antidiuretic hormone secretion (SIADH)
The cerebral oedema resulting from water intoxication causes drowsiness, lethargy, irritability, mental confusion and disorientation, with seizures and coma being
the most profound features
Peripheral oedema is remarkably rare
The patient is usually asymptomatic until the sodium level falls below 120 mmol/l, and the hyponatraemia is dilutional in type with a low serum osmolality
Urine osmolality usually exceeds 300 mosmol/kg
The commonest cancer causing this syndrome is small-cell cancer, where it is clinically obvious in 10% of cases, with subclinical involvement detectable by a
water-loading test in more than 50%
Treatment
Restriction of fluid to a daily intake of 7001000 ml may redress the hyponatraemia, but demethylchlortetracycline (demeclocycline), 6001200 mg daily, is often
highly effective, making water restriction unnecessary
Infusion of hypertonic saline is hazardous, often precipitating cardiac failure or cerebral oedema
Specialty: MRCP 1 May 2014
Question: 15
A 73-year-old man with long-standing Parkinsons disease presents to the clinic for review. He is taking a combination of a dopamine agonist, Levodopa and a
monamine oxidase inhibitor. Whilst his motor symptoms are well controlled, his wife is concerned as he is mildly confused, is awake for many hours during the night
and is suffering from visual hallucinations, coupled with delusions that she is trying to poison him. He is quiet and withdrawn when you review him in the clinic. His
mini-mental state score is 23. He is able to walk across the examination room and perform some fine motor tasks although is unable to provide a sample of
handwriting.Which of the following is the best intervention with respect to controlling his hallucinations?
A Benzhexol
B Entacapone
C Haloperidol
D Quetiapine CORRECT ANSWER
E Rivastigmine
The answer is Quetiapine - The description suggests Lewy body dementia. Management of hallucinations and agitation in association with Parkinsons disease
is difficult, with typical anti-psychotic options such as Haloperidol precipitating a significant worsening in motor function. This leaves low dose atypical
antiipsychotics such as Quetiapine as the most appropriate intervention. Rivastigmine is utilised primarily for the treatment of dementia related to Alzheimers
disease. Benzhexol may have a role in managing extra-pyramidal side effects associated with Parkinsons therapy, and Entacapone is a COMT inhibitor used
in patients who have significant symptoms from wearing off of dopamine agonists or L-dopa.
Specialty: MRCP 1 May 2014
Question: 16
A 73-year-old woman who has recently started anti-tuberculous therapy is referred to the anti-coagulation clinic as despite a large increase in her warfarin dose, her INR
had still dropped from 2.5 to 1.3.Which of the following medications is most likely to have been responsible?
A Ethambutol
B Isoniazid
C Pyrazinamide
D Rifampicin CORRECT ANSWER
E Streptomycin
The answer is Rifampicin - Rifampicin is a potent CYP450 enzyme inducer; as such it is likely to be responsible for the fall in INR seen here. Isoniazid is an
inhibitor of 2C19 and 3A4, as warfarin is metabolised by the 2C9 isoenzyme it is not likely to increase or decrease the INR. Ethambutol is primarily an
inhibitor of CYP1A2 and 2E1 and has lesser inhibitory effects on 2A6, 2C9, 2C19, 2D6 and 3A4, as such it may increase the INR rather than decrease it.
Pyrazinamide does not have significant drug interactions mediated by the p450 system. Streptomycin is the prototype aminoglycoside; as such it too does not
carry any p450 liability.
Specialty: MRCP 1 May 2014
Question: 17
A 45-year-old electrical engineer comes to the Emergency Department convinced that he has severe heart disease. He has palpitations and says that he suffers a pain
where his heart is under his left breast, normally coming on when he thinks about it, seated in his lounge. He has been investigated by the GP with an exercise tolerance
test, which was entirely unremarkable. He was extensively investigated with endoscopy and CT abdomen but the results were entirely normal one year ago. He has also
complained of chronic back pain some 5 years earlier. He is a non-smoker who drinks 5 units of alcohol per week. Clinical examination is entirely normal. His BMI is
23.Which of the following is the most likely diagnosis?
A Conversion disorder
B Depression
C Hypochondriasis CORRECT ANSWER
D Obsessive compulsive disorder
E Paroxysmal AF
The answer is Hypochondriasis - This patient has a number of features of hypochondriasis including a fear of having a serious disease. The preoccupation
persists despite appropriate medical evaluation and reassurance. This is clearly causing him significant distress, and taking into account the previous GI
symptoms, the preoccupation with illness has lasted for more than 6 months. Cognitive behavioural therapy is the intervention of choice. Conversion disorders
result in a motor or sensory deficit. Whilst depression and obsessive-compulsive disorder are seen in conjunction with hypochondriasis, there is no evidence to
suggest their presence here.
Specialty: MRCP 1 May 2014
Question: 18
A 74-year-old woman with a 3-year history of Parkinsons disease managed with Ropinirole and a small dose of Levodopa comes to the clinic with deteriorating motor
symptoms. In particular she is suffering severe dyskinesia, but does not want to reduce the dose of her Parkinsons medications for fear of increasing rigidity and tremor.
Examination in the clinic reveals mild tremor and well-controlled rigidity. She does however have significant dyskinesia.Which of the following is the most
appropriate intervention?
A Amantadine CORRECT ANSWER
B Entacapone
C Lorazepam
D Reduce Levodopa
E Reduce Ropinirole
The answer is Amantadine - This patient has levodopa-related dyskinesia. As such reducing the dose of Levodopa may improve her symptoms, although it is
likely this will also significantly worsen both tremor and rigidity. Out of the interventions listed, Amantadine reduces dyskinesia without worsening motor
function by inhibiting NMDA receptors. Entacapone is a COMT inhibitor, which is primarily used for the treatment of off phenomena. Lorazepam is a
sedative benzodiazepine and is therefore not an appropriate option here.
Specialty: MRCP 1 May 2014
Question: 19
A 55-year-old man comes to the Emergency Department. He is known to have advanced small cell lung cancer, which has proved refractory to chemotherapy. On this
occasion he has severe pain in the right upper quadrant of his abdomen. Medication includes Codeine phosphate, and Paracetamol for pain relief. Examination reveals a
BP of 155/84 mmHg, he is in pain. He has an enlarged painful liver on abdominal palpation.Investigations;
Hb 10.1 g/dl
WCC 11.2 x10
9
/l
PLT 143 x10
9
/l
Na
+
137 mmol/l
K
+
4.3 mmol/l
Creatinine 189 micromol/l
ALT 380 U/l
ALP 720 U/l
Bilirubin 45 micromol/l
USS liver 3 large metastases with surrounding oedema
Which of the following is the most appropriate next intervention?
A Oral Dexamethasone CORRECT ANSWER
B Oral Lorazepam
C Oral Morphine
D Oral Naproxen
E Subcutaneous Diamorphine
The answer is Oral Dexamethasone - The most likely cause of the pain seen here is liver capsule pain. Liver enzymes are significantly deranged and there is
evidence of oedema surrounding large metastases seen on USS. Dexamethasone will reduce oedema and significantly impact on pain, although of course this is
a short-term fix. Studies involving targeted radiotherapy suggest pain can be improved in 80% of cases, and this would be the obvious next step. Non-
steroidals can also be considered for further pain control.
Specialty: MRCP 1 May 2014
Question: 20
You are taking part in a study for a new anti-epileptic from the same chemical series as Phenytoin, but thought to be associated with fewer adverse events. The company
plans to institute therapeutic drug monitoring. What is the major reason for this in the case of Phenytoin related agents?
A CYP450 inhibition
B Hepatotoxicity at the therapeutic dose
C Interaction protein bound agents co-prescribed
D Narrow therapeutic range CORRECT ANSWER
E Nephrotoxicity at the therapeutic dose
The answer is Narrow therapeutic range - Therapeutic drug monitoring is either pharmacokinetic (measuring drug concentration), or pharmacodynamic
(measuring the effect pf the drug clinically). Phenytoin is subject to therapeutic drug monitoring because neurological toxicity is seen once the upper limit of
the therapeutic range is breached.The range of indications for therapeutic drug monitoring in general are listed below:
A relationship has been determined between plasma drug concentration and the pharmacological effect
Knowledge of the drug level influences management
There is narrow therapeutic window
There are potential patient compliance problems
The drug dose cannot be optimised by clinical observation alone
Specialty: MRCP 1 May 2014
Question: 21
A 24-year-old woman presents to the Endocrinology Clinic for review. She has a goitre, palpitations and weight loss, and has most recently developed some blurring of
vision. Her TSH is suppressed at <0.05 mU/l. On examination her BP is 132/82 mmHg, pulse is 90/min and regular. She has a fine tremor and is perspiring at rest. She
has significant bilateral proptosis with lid lag and diplopia on lateral gaze. Which of the following therapies for her Graves thyroiditis is most likely to worsen her
eye disease?
A Carbimazole
B Potassium perchlorate
C Prednisolone
D Propranolol
E Radioiodine CORRECT ANSWER
The answer is Radioiodine - Radioiodine is recognised to worsen thyroid eye disease and as such is best avoided until eye disease is stable. Interventions at this
stage for eye disease include systemic steroids as initial therapy, steroid-sparing agents may be used as may Ciclosporin, and there is emerging data to support
the use of agents such as rituximab. Carbimazole should not worsen thyroid eye disease significantly, particularly when used as part of a block replace regimen
as is usual for Graves disease. Potassium perchlorate is used as an acute treatment for thyrotoxicosis prior to emergency surgery.
Specialty: MRCP 1 May 2014
Question: 22
A 61-year-old man with a history of rheumatoid arthritis over the past 8 years comes to the clinic for review. He complains of recurrent episodes of red eyes, with mild
discomfort and grittiness, watering, and mild photophobia. The vision is normal during these episodes. The episodes occur once every few months and last for 7-10 days
each time. On this attendance at the clinic he tells you he is coming to the end of an episode. There is diffuse reddening of both eyes with engorged episcleral vessels.
Visual acuity is normal.Which of the following is the most likely diagnosis?
A Chorioretinitis
B Conjunctivitis
C Episcleritis CORRECT ANSWER
D Scleritis
E Uveitis
The answer is Episcleritis - Episcleritis carries a more benign pattern of symptoms compared with scleritis. Whilst it is less likely to be associated with systemic
disease than scleritis, it is seen in patients with a history of rheumatoid arthritis.Features of episcleritis are listed below:
Acute onset
Mild pain/discomfort, grittiness
Localised or diffuse red eye
Unilateral or bilateral
No associated ocular symptoms other than watering and occasional mild photophobia; vision normal
Features of scleritis are listed here:
Usually gradual onset
Severe boring eye pain, often radiating to forehead, brow and jaw
Pain worse with movement of the eye and at night
Localised or diffuse red eye
Associated watering, photophobia and gradual decrease in vision diplopia (posterior disease)
50% of cases are bilateral
Occasional associated systemic symptoms (fever, vomiting, headache)
Scleromalacia perforans may present with minimal symptoms
Posterior scleritis may present with very severe symptoms but a quiet white eye
Chorioretinitis is inflammation of the vascular layer of the retina, it is seen in patients with toxoplasma and CMV infection, (primarily those who are HIV
positive).
Specialty: MRCP 1 May 2014
Question: 23
A 76-year-old woman presents to the Emergency Department with bleeding around her gums when she cleans her teeth. Over the past few weeks she has been feeling
increasingly lethargic and is finding it increasingly difficult to eat because of abdominal fullness. Other past medical history of note includes two episodes of herpes
zoster over the past 6 months. Examination reveals a BP of 135/72 mmHg, pulse is 88/min and regular. She looks pale and there is evidence of dried blood around her
gums and bruising on both her arms and legs. Abdominal examination reveals gross splenomegaly. Investigations;
Hb 8.9 g/dl
WCC 62.1 x10
9
/l (lymphocytosis)
PLT 94 x10
9
/l
Na
+
137 mmol/l
K
+
4.3 mmol/l
Creatinine 103 micromol/l
ESR 85 mm 1
st
/hour
Which of the following is the most likely diagnosis?
A Chronic lymphocytic leukaemia CORRECT ANSWER
B Chronic myeloid leukaemia
C Myelodysplasia
D Myelofibrosis
E Non-Hodgkins lymphoma
The answer is Chronic lymphocytic leukaemia (CLL) - The marked lymphocytosis, massive splenomegaly and insidious onset of symptoms over a prolonged
period fits best with a diagnosis of CLL. Chlorambucil and Cyclophosphamide are usually considered for initial therapy. Rituximab has improved response
rates in combination with traditional agents, and is now approved by NICE in combination with Fludarabine and Cyclophosphamide. Most patients however
find it very difficult to cope with combination chemotherapy due to their age and functional status at diagnosis.
Specialty: MRCP 1 May 2014
Question: 24
A 45-year-old man with a known history of alcoholism presents to the Emergency Department with severe epigastric pain, nausea and vomiting. He is unable to keep
any food down and is doubled up in abdominal pain when he arrives. He drinks 5 pints of strong lager per day and a bottle of 14% red wine. Examination reveals a BP of
155/90 mmHg, pulse is 94/min and regular. He is exceptionally tender in the epigastrium, there is mild ascites, and a number of spider naevi over the upper chest. A
blood sample is lipaemic.What is the cause of the lipaemic sample in this situation?
A Chylomicrons CORRECT ANSWER
B HDL cholesterol
C IDL cholesterol
D LDL cholesterol
E VLDL cholesterol
The answer is Chylomicrons - Chylomicrons are increased acutely in patients who suffer from acute pancreatitis and lead to lipaemia noted on blood sampling.
85% of the chylomicron particle consists of triglycerides. VLDL is produced by the liver and is a precursor for LDL cholesterol. IDL is also formed by the
degradation of VLDL, and it is either then further degraded to LDL or cleared. HDL is responsible for the transport of cholesterol from peripheral tissues to
the liver. Hypertriglyceridaemia is of course a risk factor for acute pancreatitis. Familial low HDL is known as Tangier disease, and is associated with
hepatosplenomegaly and mild hypertriglyceridaemia.
Specialty: MRCP 1 May 2014
Question: 25
A 72-year-old man is referred to the Cardiology Clinic by his GP complaining of chronically increased shortness of breath and decreased exercise tolerance over the past
3-4 weeks. He has a history of ischaemic heart disease including a previous inferior MI and takes a number of medications including Aspirin, Atorvastatin, Amlodipine,
Lisinopril and Furosemide. He may have been in AF for some time and GP has already commenced warfarin therapy. On examination his BP is 155/85 mmHg, pulse is
105/min (atrial fibrillation). He is not in cardiac failure.Investigations;
Hb 13.1 g/dl
WCC 7.1 x10
9
/l
PLT 200 x10
9
/l
Na
+
138 mmol/l
K
+
4.5 mmol/l
Creatinine 108 micromol/l
ECG atrial fibrillation, old inferior Q waves
CXR cardiomegaly, prominent pulmonary vasculature
Which of the following is the most appropriate intervention?
A Amiodarone
B Bisoprolol CORRECT ANSWER
C Digoxin
D Flecainide
E Verapamil
The answer is Bisoprolol - In this situation Flecainide is contra-indicated due to the history of ischaemic heart disease. Whilst Digoxin and Verapamil slow the
ventricular rate, they are not associated with an outcome benefit. This leaves Amiodarone and Bisoprolol as possible choices. Bisoprolol is preferred, as it has
proven outcome benefits without the liabilities associated with Amiodarone (photosensitivity and thyroid disease), and will slow the ventricular rate.
Echocardiography to assess the likelihood of success of cardioversion (related to atrial size), is the next most appropriate step.
Specialty: MRCP 1 May 2014
Question: 26
A 60-year-old man presents to the Respiratory Clinic with increasing shortness of breath over the past 2 years and a dry cough. He has also had two admissions with a
lower respiratory tract infection over the past 6 months. He smokes 30 cigarettes per day and drinks 20 units of alcohol per week. Past history of a previous myocardial
infarction and rheumatoid arthritis for which he takes Methotrexate is noted. Examination reveals a BP of 123/82 mmHg, pulse is 75/min (atrial fibrillation). He has
bilateral wheeze on auscultation of the chest, with crackles at both bases. There is bilateral pitting oedema of both ankles.Investigations;
Hb 13.2 g/dl
WCC 7.9 x10
9
/l
PLT 199 x10
9
/l
Na
+
137 mmol/l
K
+
4.5 mmol/l
Creatinine 103 micromol/l
ESR 21 mm/1
st
hour
CRP 17 mg/l
CXR changes consistent with COPD, fluffy shadowing at both bases
Which of the following is the most useful next investigation?
A Arterial blood gases
B Bronchoscopy and biopsy
C CT thorax
D Echocardiogram
E Pulmonary function tests CORRECT ANSWER
The answer is Pulmonary function tests - In this situation possible diagnoses include Methotrexate-related pneumonitis and COPD. Given the chronicity of his
symptoms, coupled with the significant smoking history, and features of right heart failure, COPD is probably more likely. Pulmonary function tests will be a
helpful pointer to the underlying diagnosis, a restrictive picture seen in Methotrexate-related disease, obstructive picture in COPD, they are non-invasive and
involve no risk to the patient. Depending on the findings CT thorax or bronchoscopy may be logical next steps. The presence of isolated right heart failure is
suggestive that this is secondary to chronic respiratory disease rather than a primary cardiac problem.
Specialty: MRCP 1 May 2014
Question: 27
A 50-year-old woman comes to the Emergency Department complaining of sudden loss of colour vision and blurring affecting her left eye. There is a history of
hypertension for which she takes Amlodipine 5mg. Examination reveals a BP of 135/80 mmHg, pulse is 72/min and regular. Her BMI is 23. Fundoscopy reveals a
swollen left optic disc, and visual field testing reveals a left central scotoma.Where is the most likely site of the lesion?
A Occipital lobe
B Optic chiasm
C Optic nerve CORRECT ANSWER
D Optic radiation
E Temporal lobe
The answer is Optic nerve - Lesions in the main optic radiation or optic peduncle cause complete homonymous hemianopia without macular sparing. Lesions
in the occipital lobe lead to cortical blindness. Temporal lobe lesions result in a superior quadrantanopia. Chiasmal lesions classically produce a bitemporal
hemianopia, although if the defect is caused by pressure from above or below, the defect may be worse in the lower or upper half of the visual field
respectively.
Specialty: MRCP 1 May 2014
Question: 28
A 19-year-old man is brought to the Emergency Department by his parents. They are concerned as he has had up to 3 episodes of loss of consciousness over the past
24hrs, and during the intervening period he has become increasingly agitated. He drinks up to 9 litres of strong cider per day and is currently out of work, apparently he
has stopped drinking since running out of money and his parents have refused to supply him with alcohol. When he is reviewed in the clinic he is pale, sweating and
agitated. His BP is elevated at 152/92 mmHg, pulse is 95/min and regular. There are a number of spider naevi on examination of the upper chest. Whilst you are writing
up your findings he suffers a short generalised tonic-clonic seizure lasting for approximately 90 seconds.Investigations;
Hb 12.9 g/dl
WCC 8.9 x10
9
/l
PLT 182 x10
9
/l
Na
+
138 mmol/l
K
+
4.1 mmol/l
Creatinine 105 micromol/l
ALT 86 U/l
ALP 121 U/l
Bilirubin 16 micromol/l
Glucose 6.1 mmol/l
Which of the following is the most appropriate initial intervention with respect to the seizures?
A Carbamazepine
B Chlordiazepoxide CORRECT ANSWER
C 10% Dextrose
D Magnesium
E Pabrinex
The answer is Chlordiazepoxide - This man is almost certainly suffering from alcohol withdrawal seizures, and intervention with a benzodiazepine such as
Diazepam or Chlordiazepoxide is required to prevent further seizures. Of course thiamine deficiency often co-exists with alcohol withdrawal, and
administration of Pabrinex A and B is therefore also advised. In the case of significant hypoglycaemia, glucose replacement with 10% dextrose is required,
although in this situation co-prescription of thiamine replacement with the glucose is essential. Magnesium may be of value in patients with resistant seizures or
rhythm disturbance, particularly as magnesium diuresis is a recognised feature of alcohol abuse.
Specialty: MRCP 1 May 2014
Question: 29
A 24-year-old woman with a history of hereditary spherocytosis (HS) presents to the Emergency Department with right upper quadrant pain and vomiting. This is her
third episode of right upper quadrant pain to occur over the past 6 months. Apart from the HS she is otherwise well, and her only medication of note is the combined oral
contraceptive pill. Examnation reveals a temperature of 37.8C, pulse is 95/min and regular, BP is 155/100 mmHg. She is in pain at rest and very tender in the right
upper quadrant and epigastrium on palpation. Bowel sounds are present.Investigations;
Hb 13.4 g/dl
WCC 12.4 x10
9
/l
PLT 210 x109/l
Na
+
137 mmol/l
K
+
4.5 mmol/l
Creatinine 110 micromol/l
ALT 85 U/l
ALP 284 U/l
Bilirubin 34 micromol/l
Amylase 61 U/l
You suspect acute cholecystitis related to gallstones.What is the most likely composition of her gallstones?
A Black pigment CORRECT ANSWER
B Brown pigment
C Calcium
D Cholesterol
E Mixed
The answer is Black pigment - Bile contains a number of substances including cholesterol, bile pigments from metabolised haemoglobin, and phospholipids.
According to differences in aetiology, different gallstones are seen.
Cholesterol stones, which make up 80% of all GB stones in the UK, are large, often solitary and radiolucent. They are seen in association with
hypertriglyceridaemia
Black pigment stones are small, friable, irregular and radiolucent and are comprised largely of bile pigments. Risk factors are those for haemolysis, (i.e.,
sickle cell anaemia, hereditary spherocytosis, thalassaemia) and cirrhosis
Mixed stones are faceted stones and are comprised of calcium salts, pigment and cholesterol. 10% of these are radiopaque and may therefore be seen on
plain x-ray
Brown pigment stones which are very rarely seen in the UK form as a result of stasis and infection within the biliary system, usually in the presence of
escherichia coli and/or klebsiella
Specialty: MRCP 1 May 2014
Question: 30
A 45-year-old man presents with progressive fatigue and loss of energy, which has increased significantly over the last 4 months. He has a history of Hodgkins
lymphoma for which he received chemoradiotherapy some 18 years earlier. In the intervening years he has been well, but reports slowing down, with an inability to do
sport over the past 2 years or so. His BP is 145/82 mmHg, pulse is 88/min and regular. JVP is elevated and there is a steep y descent. There are no murmurs, he has
bilateral pitting oedema of both ankles. Abdominal palpation reveals significant pulsatile hepatic enlargement. Routine screening bloods including ESR are
unremarkable. Investigations:ECG Low voltage complexes CXR Normal sized cardiac shadow, bilateral small pleural effusionsWhich of the following is the most
likely diagnosis?
A Dilated cardiomyopathy
B Malignant pericarditis
C Post-radiation constrictive pericarditis CORRECT ANSWER
D Post-radiation pulmonary fibrosis
E Primary pulmonary hypertension
The answer is Post-radiation constrictive pericarditis - Constrictive pericarditis may present many years after the original insult (in this case the radiotherapy
given some 18 years earlier). In many cases the cardiac shadow looks normal on CXR, pericardial calcification is only seen in some 20-30%. Raised JVP with
steep y descent, pulsatile liver and other signs of right heart failure are typical of the condition. In this case pericardectomy is likely to be required. Given the
lack of chest signs significant pulmonary fibrosis is unlikely. The presence of normal inflammatory markers counts against malignant pericarditis although of
course it does not rule it out completely. Cardiac enlargement would be expected in dilated cardiomyopathy.
Specialty: MRCP 1 May 2014
Question: 31
A 62-year-old man presents with central crushing chest pain, palpitations and a syncopal episode, which occurred in the local supermarket. He has a history of ischaemic
heart disease and has suffered a previous inferior myocardial infarction. He arrives in the Emergency Department with a broad complex tachycardia and a pulse
approaching 200 BPM on the monitor. His BP is 95/60 mmHg.Which of the following would prompt you to think this is VT rather than SVT with aberrant
conduction if you saw it on the 12 lead ECG?
A Absence of fusion beats
B AV dissociation CORRECT ANSWER
C Normal ECG axis
D QRS 150 ms
E Rate <220 BPM
The answer is AV dissociation - A number of features can be identified with respect to the ECG, which increase the likelihood of VT. These include:
Absence of typical RBBB or LBBB morphology
Extreme axis deviation QRS is positive in aVR and negative in I + aVF
Very broad complexes (>160ms)
AV dissociation (P and QRS complexes at different rates)
Capture beats occur when the sinoatrial node transiently captures the ventricles, in the midst of AV dissociation, to produce a QRS complex of
normal duration
Fusion beats occur when a sinus and ventricular beat coincides to produce a hybrid complex
Positive or negative concordance throughout the chest leads
Brugadas sign The distance from the onset of the QRS complex to the nadir of the S-wave is > 100ms
Josephsons sign Notching near the nadir of the S-wave
RSR complexes with a taller left rabbit ear. This is the most specific finding in favour of VT. This is in contrast to RBBB, where the right rabbit ear is
taller
Specialty: MRCP 1 May 2014
Question: 32
A 22-year-old woman presents with severe low mood some 14 days after the birth of her first child. She says that she has not slept since a few days after the birth,
waking at approximately 4am, even if her partner takes the baby for overnight feeds. She is breastfeeding during the day. She is not eating and has lost 4kg in weight,
and feels that she is worthless. She tells you that it might be better for everyone if she just goes away, and cries during the course of the consultation. There are no
abnormal physical findings.Which of the following is the most appropriate intervention?
A Cognitive behavioural therapy
B Citalopram
C Doxepin
D Progesterone only pill
E Sertraline CORRECT ANSWER
The answer is Sertraline - In this situation the patient is showing significant signs of depression; cognitive behavioural therapy is therefore likely to be
insufficient with respect to managing symptoms over the short term. Fluoxetine, Citalopram and Escitalopram are not recommended in patients who may
choose to breast feed because of significant accumulation in breast milk. Sertraline is present at much lower levels in breast milk vs these other SSRIs. Doxepin
on the other hand is not recommended in breast-feeding. Cognitive behavioural therapy could of course be used as an adjunct to therapy in this patient.
Specialty: MRCP 1 May 2014
Question: 33
A 19-year-old woman comes to the Sleep Clinic for review. She complains of excessive daytime sleepiness such that she feels she is unable to keep up with her
university course. In addition on two separate occasions over the past few months, she has collapsed appearing to be unconscious, without warning, on one occasion
whilst in animated debate with friends, on the other occasion whilst laughing at a movie. She has no past medical history, her only medication is the combined oral
contraceptive pill. She drinks 10 pints of beer per week and smokes 5 cigarettes per day. Examination reveals a BP of 115/80 mmHg, pulse is 65/min and regular. No
neurological abnromalities are identified. Her BMI is 23.Investigations;
Hb 13.0 g/dl
WCC 7.2 x10
9
/l
PLT 212 x10
9
/l
Na
+
137 mmol/l
K
+
4.1 mmol/l
Creatinine 89 micromol/l
Glucose 5.1 mmol/l
12 lead ECG normal
Given the suspected diagnosis, which of the following is the intervention most likely to relieve day time sleepiness?
A Alcohol reduction
B CPAP
C Modafinil CORRECT ANSWER
D Sodium valproate
E Weight loss
The answer is Modafinil - The history here fits best with narcoplepsy (the excessive day time sleepiness), and catoplexy (the episodes of collapse). Unlike in
obstructive sleep apnoea, CPAP or weight loss are not likely to significantly impact on symptoms. Reduction of excessive alcohol intake may impact on
symptoms, although 10 pints of beer per week is unlikely to be a significant contributor to sleepiness. Sleep hygeine measures such as avoiding stimulants
including caffeine prior to bed, regular exercise, and planning for 7-8hrs sleep per night may also be of benefit.
Specialty: MRCP 1 May 2014
Question: 34
A 22-year-old traveller presents to the Emergency Department with acute jaundice. She has suffered from increasing nausea and flu-like symptoms over the past few
days with darkening of her urine, and now feels unable to eat. She admits to intermittent use of IV heroin and has had unprotected intercourse on a number of occasions
over the past 5 years. Only medication of note is the combined oral contraceptive pill. She is clearly jaundiced, her temperature is 37.5C and her BP is 105/70 mmHg,
pulse is 75/min and regular. Her abdomen is soft although she is tender in the right upper quadrant.Investigations;
Hb 13.1 g/dl
WCC 11.9 x10
9
/l
PLT 199 x10
9
/l
Na
+
137 mmol/l
K
+
4.9 mmol/l
Creatinine 110 micromol/l
ALT 1210 U/l
Bilirubin 85 micromol/l
ALP 295 U/l
Anti HBs IgG positive
Anti HBc IgG positive
HB e antigen negative
Anti Hep A IgM positive
Anti Hep C IgG positive
Which of the following is the most likely diagnosis?
A CMV infection
B Hepatitis A CORRECT ANSWER
C Hepatitis B
D Hepatitis C
E Hepatitis E
The answer is Hepatitis A - The acute symptoms seen here with a short prodromal illness and later jaundice fit best with a diagnosis of hepatitis A. The Hep A
IgM status is also consistent with hepatitis A. The hepatitis B and C serology potentially indicates previous exposure to both viruses. The picture is not
consistent with hepatitis B because hep B is only very rarely associated with acute hepatitis, and the presence of IgG antibodies with negative e antigen does not
fit with acute infection. In the absence of the hep A serology we are provided with, hepatitis E would be an alternative diagnosis.
Specialty: MRCP 1 May 2014
Question: 35
A 27-year-old man who has been walking in the Austrian Alps comes to the Emergency Department complaining of fevers, lethargy and joint pains and most recently a
left facial nerve palsy. He remembers suffering from a tick bite on his left shin and has some residual rash in the area. Examination reveals a temperature of 37.9C, his
BP is 122/72 mmHg, pulse is 62/min and regular. He has a left facial nerve palsy and there is a fading, annular, erythematous rash affecting his left shin. He has been
started on a course of Ciprofloxacin by a local GP in Austria.Which of the following antibiotics would be the most appropriate to change to?
A Cephalexin
B Clarithromycin
C Co-amoxiclav
D Doxycycline CORRECT ANSWER
E Penicillin V
The answer is Doxycycline - Over the past few years, resistant strains of borrelia (largely mediated by changes in topoisomerase IV), have begun to emerge. As
such Doxycycline is a better choice with respect to therapy for Lyme disease. Alternatives include Amoxicillin, Co-amoxiclav and Cefuroxime. A two-week
course of oral therapy is advised. Prognosis for neurological recovery is good, even in those patients who are diagnosed and treated late with antibiotic therapy.
Specialty: MRCP 1 May 2014
Question: 36
A 23-year-old man presents to the Rheumatology Clinic. He has pain in the lower back and sacroiliac region, which has been increasing in severity over the past six
months. He says that when he gets out of bed in the morning he can hardly move, only after a hot shower and a walk of some 2km to work does his back loosen up and
finally begin to improve. He has also suffered from plantar fasciitis on two occasions over the past 3 years. Examination reveals limitation of forward and lateral flexion
of the lumbar spine due to pain and stiffness. Lower limb neurological exam is entirely normal. Investigations;
Hb 13.2 g/dl
WCC 8.0 x10
9
/l
PLT 203 x10
9
/l
Na
+
136 mmol/l
K
+
4.5 mmol/l
Creatinine 103 micromol/l
ESR 62 mm/1
st
hour
Lumbar spine X-ray blurring of the lower portion of the sacroiliac joints bilaterally
Which of the following is the most likely diagnosis?
A Ankylosing spondylitis CORRECT ANSWER
B Osteoarthritis
C Prolapsed lumbar disc
D Rheumatoid arthritis
E Spinal stenosis
The answer is Ankylosing spondylitis (AS) - Chronic lumbar back pain coupled with other evidence of enthesitis (plantar fasciitis) and early sacroilitis as
evidenced on the lumbar spine x-ray is suggestive of a diagnosis of AS. MRI scanning may be useful in determining the extent of inflammation as it has greater
sensitivity vs plain x-ray alone for early changes. Early aggressive therapy is now recognised to prevent spinal deformity, and intervention with TNF-alpha
receptor modulating drugs is indicated after the failure of two different NSAIDs to control symptoms. There are no lower limb symptoms to suggest a
prolapsed lumbar disc. Lumbar spinal stenosis is associated with bilateral lower limb pain, worse after a period of exercise and relieved by rest.
Specialty: MRCP 1 May 2014
Question: 37
A 58-year-old man returns to the Emergency Department with 30 mins of central chest pain radiating to the upper part of his left arm. He suffered an inferior territory
NSTEMI some 6 months earlier with a small troponin rise to 1.2 and current cardiovascular medication includes Lisinopril, Bisoprolol, Aspirin and Atorvastatin. His
pain is relieved by 2 sprays of GTN. Physical examination is unremarkable.Investigations;
Hb 13.1 g/dl
WCC 8.1 x10
9
/l
PLT 213 x10
9
/l
Na
+
138 mmol/l
K
+
4.3 mmol/l
Creatinine 102 micromol/l
HbA1c 55 mmol/mol (7.2%)
Random glucose 6.7 mmol/l
12 lead ECG normal sinus rhythm, inferior Q waves
Which of the following is the most appropriate way to manage him?
A Prophylactic LMW Heparin and observe overnight
B Thrombolysis
C Treatment dose LMW Heparin and observe overnight
D Treatment dose Fondaparinux, Aspirin and Clopidogrel CORRECT ANSWER
E Urgent angiography
The answer is Treatment dose Fondaparinux, Aspirin and Clopidogrel - Despite the fact this patient has no acute changes on his ECG, he should be managed
aggressively for ACS, as it is quite possible that the 6hr troponin will be elevated. Given that he has had an NSTEMI 6 months earlier, he is at significant risk
of a further infarct and should be considered for angiography, electively within a few weeks if the pain settles on this occasion, or as an inpatient if the pain
fails to settle.
Specialty: MRCP 1 May 2014
Question: 38
A 32-year-old shipyard worker presents to the Respiratory Clinic for review. He has suffered a third episode of right-sided pneumonia over the past 4 months and is
concerned that he is not completely recovering between each episode. He has a history of recurrent pneumonia episodes in childhood, asthma, and feels like he
constantly has a chest infection. Apparently each morning he spends up to 20 mins trying to cough up purulent sputum even when well. On examination his BP is 122/72
mmHg, pulse is 85/min and regular. He is apyrexial. There is scattered wheeze on auscultation and crackles at the right base. Investigations;
Hb 13.1 g/dl
WCC 11.2 x10
9
/l
PLT 231 x10
9
/l
Na
+
137 mmol/l
K
+
4.3 mmol/l
Creatinine 112 micromol/l
CRP 175 mg/l
CXR right lower lobe consolidation
PFTs (tested when well some 2 months earlier) mixed obstructive / restrictive defect
Which of the following is the most likely underlying diagnosis?
A Asbestosis
B Asthma
C Bronchiectasis CORRECT ANSWER
D Bronchial carcinoid
E COPD
The answer is Bronchiectasis - The episodes of recurrent pneumonia since childhood, coupled with a mixed obstructive / restrictive defect on CXR and chronic
cough productive of sputum even when well, raise the possibility of bronchiectasis. This may be related to underlying immunoglobulin deficiency or an
infection in childhood such as whooping cough or chicken pox pneumonitis, which increase the risk of long-term bronchiectasis. Exposure to asbestos in
modern shipbuilding is minimal, and bronchial carcinoid is a vascular tumour that usually leads to haemoptysis. Immunoglobulin electrophoresis is a logical
next step in investigating possible deficiency.
Specialty: MRCP 1 May 2014
Question: 39
A 32-year-old man presents to the clinic for review. He has suffered from profuse watery diarrhoea over the past few months and has multiple negative stool cultures.
He works as a taxi driver but has been forced to go on long term sick leave because of the problem. He says the diarrhoea is profuse and watery and does not seem to be
related to food. He has lost a few kg in weight over the past 2 months, and feels thirsty all the time, drinking copious amounts of water to keep up with his GI losses. On
examination his BP is 105/60 mmHg, with a postural drop of 15mmHg, pulse is 85/min and regular. His abdomen is soft and non tender, his BMI is 22, PR reveals an
empty rectum. Investigations;
Hb 12.2 g/dl
WCC 8.9 x10
9
/l
PLT 203 x10
9
/l
Na
+
136 mmol/l
K
+
3.1 mmol/l
Chloride 119 mmol/l
Bicarbonate 14 mmol/l
Anti-endomyseal antibody negative
Faecal fat 3.9 g/24hrs
Stool volume 3.5 l
Which of the following is likely to be the most useful next investigation?
A Plasma VIP CORRECT ANSWER
B 24hr catecholamines
C 24hr urinary 5-HIAA
D D-xylose absorption test
E Hydrogen breath test
The answer is Plasma VIP - The watery diarrhoea, coupled with a normal anion gap metabolic acidosis is very suggestive of VIPoma. Levels of VIP are usually
between 2 and 10 times the normal range. The anti-endomyseal antibody and faecal fat results count against significant malbsorption. 24hr catecholamines are
considered in suspected phaeochromocytoma, 5-HIAA in carcinoid. D-xylose absorption is abnormal in diseases which result in intestinal malabsorption such
as Whipples, and hydrogen breath testing is usually considered for diagnosing bacterial overgrowth syndrome.Contrast CT of the abdomen or MRI are both
modalities considered for localisation of the primary tumour within the pancreas. Tumours are usually large, (3cm or greater in diameter) at the point of
diagnosis.
Specialty: MRCP 1 May 2014
Question: 40
A 62-year-old man presents to the Emergency Department with lower back pain, which began whilst he was lifting boxes helping his daughter move house. He has mild
asthma for which he takes a low dose seretide inhaler, and hypertension controlled with Nifedipine MR, but no other significant past medical history. On examination his
BP is 138/82 mmHg, pulse is 85/min and regular and he looks in pain. Neurological examination reveals lack of sensation most marked on the lateral aspect of the
dorsum of the left foot, and the bottom of the left foot and weakness of plantar flexion. Ankle reflexes are diminished bilaterally.Which of the following is the nerve
root most likely to be involved?
A L2
B L3
C L4
D L5
E S1 CORRECT ANSWER
The answer is S1 - The history is most consistent with an acute lumbosacral disc prolapse related to the lifting activity. The pattern of sensory and motor
involvement fits best with a predominant S1 lesion. The list of lower limb dermatomes and myotomes is provided below:Points characteristically ascribed to
each dermatome: L1: Midway between the key sensory points for T12 and L2. L2: Anterior medial thigh, midpoint of a line between the midpoint of the
inguinal ligament and the medial epicondyle of the femur. L3: Medial epicondyle of the femur. L4: Over the medial malleolus. L5: On the dorsum of the foot at
the third metatarsophalangeal joint. S1: Lateral aspect of the calcaneus.L2: hip flexion L3: knee extension L4: ankle dorsi-flexion L5: great toe extension S1:
ankle plantar-flexion/ankle eversion/hip extension
Specialty: MRCP 1 May 2014
Question: 41
A 71-year-old woman presents to the Dermatology Clinic with a flat, pigmented area on her left cheek. She says the lesion has been there for some time but has recently
grown, and it is clear she has extensive sun damage to her face, legs and arms from living in South Africa in the past. Examination reveals extensive small, pigmented
lesions consistent with sun exposure. The largest of these is flat, is 2cm in diameter and has an irregular pigmented border. Which of the following is the most likely
diagnosis?
A Basal cell carcinoma
B Lentigo maligna CORRECT ANSWER
C Pityriasis vesicolor
D Solar keratosis
E Squamous cell carcinoma
The answer is Lentigo maligna - The pigmented, flat nature of the lesion here, against a background of sun-damaged skin, is most consistent with lentigo
maligna, a pre-invasive lesion, which has the potential to develop into malignant melanoma. The ideal intervention is surgical excision, although in patients
where this is not possible, cryotherapy and topical immunotherapy are possible alternatives. Solar keratoses and squamous cell carcinoma are associated with
erythematous, scaling skin. Basal cell carcinoma is associated with skin thickening and ulceration. Pityriasis vesicolor usually results in an annular,
erythematous scaling rash on the trunk.
Specialty: MRCP 1 May 2014
Question: 42
A 19-year-old man with Ehlers-Danlos Type IV comes to the Emergency Department because of acute blindness affecting his right eye. He had normal vision in both
eyes until he awoke on the morning of presentation, where his vision had deteriorated to count fingers. Examination of the left eye is normal. Examination of the right
eye via direct fundoscopy reveals a blurring of the retina with a large central area that appears grey in colour. Which of the following is the most likely diagnosis?
A Lens dislocation
B Optic neuritis
C Retinal artery occlusion
D Retinal vein occlusion
E Retinal detachment CORRECT ANSWER
The answer is Retinal detachment - Unlike Marfans syndrome, where lens dislocation is seen, vascular type Ehlers-Danlos (Type IV) is associated with retinal
detachment because of haemorrhage. Patients are also often short sighted. Ehlers Danlos is not associated with a marked increase in atherothrombosis; so
retinal vein occlusion and artery occlusion are less likely options here. These options and optic neuritis are not associated the retinal changes seen here. Median
lifespan for Ehlers Danlos type IV is 48 years, with death occurring usually due to rupture of medium or large arteries; rupture of the sigmoid colon has also
been reported.
Specialty: MRCP 1 May 2014
Question: 43
A 32-year-old woman comes to the Emergency Department suffering from a high fever, flu-like symptoms and a generalised maculopapular rash. She had commenced
treatment 24hrs earlier with oral Doxycycline having suffered a tick bite to her right shin whilst walking in mountains in Southern Germany. On examination she is
pyrexial 38.4C, her BP is 122/72 mmHg; pulse is 78/min and regular. There Is an annular erythematous rash around her original tick bite, and a fine, generalised
maculopapular rash elsewhere. Investigations;
Hb 13.1 g/dl
WCC 10.8 x10
9
/l
PLT 203 x10
9
/l
Na
+
138 mmol/l
K
+
4.3 mmol/l
Creatinine 100 micromol/l
Which of the following is the most likely diagnosis?
A Allergy to Doxycycline
B Concurrent viral illness
C Jarisch Herxheimer reaction CORRECT ANSWER
D Stevens Johnson syndrome
E Treatment failure
The answer is Jarisch Herxheimer (JH) reaction - In a similar picture to that seen in syphilis, there is an increased risk of the JH reaction occurring shortly
after the commencement of antibiotic therapy due to death of the borrelia spirochetes. In most cases no intervention is required. Non-steroidals and
corticosteroids may be considered, although they are not thought to impact on clinical outcomes. Stevens Johnson (SJ) syndrome begins as a macular
erythematous rash, but quickly spreads to involve the mucosal surfaces, the lesions breaking down with an ulcerated and/or necrotic core. SJ syndrome is seen
more rarely with Tetracyclines than with some of the other antibiotic classes. An allergic rash related to Doxycycline is a potential alternative diagnosis, but the
timing of the rash in the context of borrelia infection, fits very well with the JH reaction.
Specialty: MRCP 1 May 2014
Question: 44
A 56-year-old woman is referred to the Neurology Department for assessment of a movement disorder. According to the GP she has developed features of Parkinsons
disease and a number of falls over the last 6 months, and she has not had a marked response to Dopamine agonist therapy. There is a past history of hypertension for
which she takes Amlodipine 5mg daily, but nil else of note. Examination reveals a BP of 155/95 mmHg, with a postural drop of 30mmHg on standing; her pulse is
70/min and regular. She has bilateral rigidity (worse proximally than distally), and tremor, although the tremor is less than you might expect given how rigid she
appears. There is loss of upward gaze. When you ask her to walk there is significant postural instability. She also has significant impairment of verbal fluency whilst
trying to explain her symptoms to you and finds it difficult to swallow a glass of water.Investigations;
Hb 13.1 g/dl
WCC 6.5 x10
9
/l
PLT 198 x10
9
/l
Na
+
138 mmol/l
K
+
4.5 mmol/l
Creatinine 110 micromol/l
Glucose 8.1 mmol/l
CT head no focal changes
Which of the following is the most likely diagnosis?
A Corticobasal degeneration
B Idiopathic Parkinsons disease
C Multi-system atrophy
D Normal pressure hydrocephalus
E Progressive supranuclear palsy CORRECT ANSWER
The answer is Progressive supranuclear palsy (PSP) - The features suggestive of a diagnosis of PSP in this case include:
Falls seen very shortly after the onset of symptoms
Symmetrical (proximal greater than distal) rigidity
Poor response to levodopa therapy
Dysphagia
Decreased verbal fluency
Corticobasal degeneration usually presents with muscle spasms, rigidity and loss of control of one limb only during the initial period. As it progresses speech
becomes slow and slurred, falls become more frequent and dementia ensues. Whilst some autonomic dysfunction is seen in Parkinsons, it is not usually as
marked early in the disease as is seen here, and loss of upward gaze is not usually seen. Normal pressure hydrocephalus presents with memory loss, gait
disturbance and incontinence. Multi-system atrophy is associated with a wider range of autonomic features than postural hypotension alone, including loss of
heart rate variability and other symptoms such as loss of sweating.
Specialty: MRCP 1 May 2014
Question: 45
You review a 54-year-old woman who has long-standing rheumatoid arthritis.Which one of the following are common features of rheumatoid arthritis?
A Ulcerative colitis
B Uveitis
C Proteinuria from renal deposition of amyloid
D A monarticular picture
E Proximal interphalangeal joint involvement in the hands CORRECT ANSWER
Features associated with rheumatoid arthritis
Rheumatoid arthritis commonly presents with a polyarticular picture, the metacarpophalangeal and the proximal interphalangeal joints are the usual sites of
involvement in the hands.
Scleritis and episcleritis are more common ocular manifestations of rheumatoid arthritis than uveitis and are usually managed with local and/or systemic
corticosteroids.
Disease-modifying agents used for treating rheumatoid arthritis are a more common cause of proteinuria in rheumatoid arthritis than amyloid deposition.
Ulcerative colitis and inflammatory bowel disease are associated with seronegative arthritides, not rheumatoid arthritis.
Specialty: MRCP 1 May 2014
Question: 46
A 60-year-old woman who had a left breast lumpectomy some 20 years ago presents to the Emergency Department with backache. She has also been increasingly thirsty,
and has been waking twice per night to pass urine over the past few weeks. Other past medical history of note includes hypertension and diabetes which is controlled
with diet and exercise only. BP is 142/72 mmHg, pulse is 80/min and regular. She has pain over the lumbar spine and flexion is limited by pain, there is no abnormal
neurology on lower limb examination.Investigations;
Hb 10.5 g/dl
WCC 8.1 x10
9
/l
PLT 178 x10
9
/l
Na
+
138 mmol/l
K
+
4.5 mmol/l
Creatinine 132 micromol/l
Albumin 29 g/l
Ca
++
2.89 mmol/l
Urine protein ++
Lumbar spine x-ray possible lytic lesions
Bone scan: no significant increase in lumbar spine uptake
Which of the following is the next most appropriate investigation?
A CT abdomen
B Immunoglobulin electrophoresis CORRECT ANSWER
C Mammogram
D MRI spine
E Renal biopsy
The answer is Immunoglobulin electrophoresis - Whilst it is not impossible for this patient to suffer a recurrence of her breast cancer, it is less likely given that
her lumpectomy took place 20 years earlier. The presence of hypercalcaemia, hypoalbuminaemia and proteinuria, coupled with changes on x-ray that are not
confirmed on bone scan, is highly suggestive of an underlying diagnosis of malignant myeloma. The optimal next investigation therefore is immunoglobulin
electrophoresis, which is likely to demonstrate a paraprotein band.
Specialty: MRCP 1 May 2014
Question: 47
A 19-year-old woman comes to the clinic for review. She complains of problems with facial and upper chest hair and severe acne. She only has a menstrual period
approximately every 3-4 months and when these do occur they are heavy. She takes no regular medications and does not currently have a sexual partner. On examination
her BP is 152/90 mmHg;, pulse is 78/min and regular. Respiratory examination is unremarkable. Her abdomen is soft and non-tender, BMI is elevated at 31. She has
facial hair, hair over her upper body, and acne affecting her face and upper chest.Investigations;
Hb 13.2 g/dl
WCC 8.1 x10
9
/l
PLT 201 x10
9
/l
Na
+
138 mmol/l
K
+
4.3 mmol/l
Creatinine 95 micromol/l
Glucose 6.5 mmol/l
LH:FSH ratio 2.2
Testosterone 5.1 nmol/l
Her main complaint is of hirsuitism and acne. Which of the following is the most appropriate initial intervention?
A Clomifene
B Combined OCP
C Co-cyprindiol (cyproterone acetate and ethinylestradiol) CORRECT ANSWER
D Metformin
E Pioglitazone
The answer is Co-cyprindiol (cyproterone acetate and ethinylestradiol) - The diagnosis here is polycystic ovarian syndrome (PCOS). Co-cyprindiol
(cyproterone acetate and ethinylestradiol; brand name Dianette) blocks androgen receptors (via the cyproterone component), and reduces androgen
production via negative feedback because of the presence of ethinyloestradiol. It is particularly effective in reducing androgenic acne and is therefore the
logical choice here. Weight loss is extremely important in the management of PCOS and the patient should also be encouraged to follow intensive life style and
diet intervention. Metformin promotes a small amount of weight loss and improves insulin sensitivity; as such it is often considered the first line intervention in
women who are trying for a child. Clomifene stimulates ovulation. Pioglitazone improves ovarian insulin sensitivity and restores ovulation, although it has
multiple side effects including fluid retention, weight gain and loss of bone mineral density and as such it is not recommended.
Specialty: MRCP 1 May 2014
Question: 48
A 42-year-old motorcyclist comes to the clinic for review. He is suffering from weakness affecting his left arm. He suffered a fracture of his humerus, which required
nailing some 4 months earlier. On examination there is weakness of extension of the forearm at the elbow, supination, wrist extension and thumb abduction. There is
sensory loss affecting the dorsal aspect of the forearm and the lateral portion of the hand, although the little finger is spared.Which of the following is the most likely
site of the lesion?
A C5/C6 nerve root
B C8/T1 nerve root
C Median nerve
D Radial nerve CORRECT ANSWER
E Ulnar nerve
The answer is Radial nerve - Radial nerve lesions are recognised to occur when trauma occurs to the upper arm, including with humeral nailing after unstable
fracture. Transient radial nerve palsy also occurs for example when patients fall asleep with their arm outstretched over a chair because of nerve
compression.Motor features are summarised below:
C7, C8: triceps
C5, C6: brachioradialis
C6, C7: extensor carpi radialis longus
C5, C6: supinator
C7, C8: extensor digitorum
C7, C8: extensor carpi ulnaris
C7, C8: abductor pollicis longus
C7, C8: extensor pollicis brevis
C7, C8: extensor pollicis longus
Sensory features include:Loss of sensation over the dorsal aspect of the forearm from below the elbow down over the lateral part of the hand to include the
thumb to the IP joint and the fingers to the distal interphalangeal (DIP) joint. It usually includes the lateral side of the ring finger but may include all or none
of it.The ulnar nerve supplies:
Flexor carpi ulnaris
Flexor digitorum profundus (medial half)
Hypothenar muscles
Adductor pollicis
The third and fourth lumbrical muscles
Dorsal interossei
Palmar interossei
Palmaris brevis
Median nerve lesions lead to weakness of hand grip and opposition of the thumb.
Specialty: MRCP 1 May 2014
Question: 49
A 23-year-old woman presents to the clinic for review. She works as a care assistant in a local nursing home. She suffered an acute attack of shortness of breath with
stridulous breathing, erythema, and a hives-type rash affecting her neck and upper chest whilst visiting the dentist, and had to be taken to the Emergency Department
where she required anti-histamines, corticosteroids and adrenalin. There have been two previous incidences. Only past history of note is of flexural eczema as a child.
Examination in the clinic is entirely normal apart from some minor eczema at the back of each knee. Which of the following is the most likely underlying
pathophysiology?
A C1 esterase deficiency
B C4 deficiency
C Type 1 hypersensitivity CORRECT ANSWER
D Type 2 hypersensitivity
E Type 4 hypersensitivity
The answer is Type 1 hypersensitivity - The likely underlying diagnosis here is latex hypersensitivity, and Type 1 hypersensitivity is responsible for the acute
anaphylaxis type response seen. It is caused by an IgE mediated response to specific latex proteins. IgE cross linking by latex proteins coupled with mast and
basophil cell interaction is the trigger for the response. Type 4 hypersensitivity is responsible for contact allergic dermatitis, which appears some 24-48hrs after
latex exposure. Type 2 hypersensitivity occurs when antibodies driven by the immune response recognise cell surface antigens. C1 esterase deficiency is not
associated with hives and does not fit with the history of flexural eczema. C4 deficiency is associated with increased risk of SLE.
Specialty: MRCP 1 May 2014
Question: 50
A 32-year-old man comes to the clinic for review. He has significant arthritis with joint calcification; he has a bluish discoloration of his ear cartilage and passes urine
that turns dark on standing. Other past medical history of note includes 2 episodes of lumbar disc herniation the pain from which had settled over a number of weeks.
Which of the following is the most likely underlying defect?
A Alpha spectrin deficiency
B Glycogen synthase deficiency
C Glucuronyltransferase deficiency
D Homogentisic oxidase deficiency CORRECT ANSWER
E Uroporphyrinogen decarboxylase deficiency
The answer is Homogentisic oxidase deficiency - This patient has an autosomal recessive condition, homogentisic acid oxidase deficiency, which leads to
alkaptonuria. It results from a set of inherited alleles on chromosome 3 (3q21-q23). It results primarily in symptoms of arthritis, and later cardiovascular
disease. Homogentisic acid is deposited in cartilage as a polymer and forms a black pigment layer. Attempts have been made to retard progression by
restricting protein in the diet, although long-term data is not robust. Glycogen synthase deficiency leads to symptoms of muscle cramps, particularly on
exercise. Uroporphyrinogen decarboxylase deficiency results in the development of porphyria; alpha spectrin deficiency hereditary spherocytosis;
glucuronyltransferase deficiency results in Gilberts syndrome.
Specialty: MRCP 1 May 2014
Question: 51
A 45-year-old man with a long history of Crohns disease presents to the Gastroenterology Clinic with lethargy and progressively increasing generalised itching over the
past 6 months. His Crohns is quiescent and he is maintained on a small dose of Mesalazine only. He opens his bowels normally 2-3 times per day. Examination reveals
a BP of 110/72 mmHg; pulse is 70/min and regular. Abdomen is soft and non-tender, apart from some vague pain in the right upper quadrant. His BMI is 22. There are
generalised scratch marks and he has jaundiced sclerae.Investigations;
Hb 10.9 g/dl
WCC 8.0 x10
9
/l
PLT 181 x10
9
/l
Na
+
137 mmol/l
K
+
4.3 mmol/l
Creatinine 115 micromol/l
ALT 85 U/l
AKP 300 U/l
Bilirubin 33 micromol/l
Glucose 5.1 mmol/l
Which of the following is the most likely diagnosis?
A Ascending cholangitis
B Chronic active hepatitis
C Primary biliary cirrhosis
D Primary sclerosing cholangitis CORRECT ANSWER
E Steatohepatitis
The answer is Primary sclerosing cholangitis (PSC) - PSC occurs in patients with a history of inflammatory bowel disease (IBD), whilst the pathophysiology is
not clear, it is thought to be autoimmune in origin and may be down to an abnormal immune response to invading organisms, which are able to cross the bowel
mucosa in IBD. As here a predominantly obstructive picture in terms of changes in LFTs is seen. IgM may be raised and pANCA, anti-cardiolipin and ANA
positivity is also seen. Magnetic resonance cholangiopancreatography (MRCP) or endoscopic retrograde cholangiopancreatography (ERCP) are the best
options for imaging and show intrahepatic bile duct structuring and dilatation. Primary biliary cirrhosis is an alternative diagnosis, although it is commoner in
women and is not necessarily associated with IBD. Steatohepatitis is seen primarily in the obese population. Ascending cholangitis presents acutely with
jaundice, pyrexia and RUQ pain.
Specialty: MRCP 1 May 2014
Question: 52
A 22-year-old woman with a diagnosis of Turners syndrome comes to the clinic for review. She enquires about her long-term future.Which of the following are
patients with Turners syndrome most at risk of?
A Diabetes mellitus
B Gynaecological cancer
C Premature cardiovascular death CORRECT ANSWER
D Renal failure
E Thyroid cancer
The answer is Premature cardiovascular death - Turners is associated with significant cardiovascular pathology including aortic coarctation, bicuspid aortic
valve, and aortic root dissection. There is also a 2-3 fold increase in hypertension in the Turners syndrome population. Given the risk of aortic root dissection,
aggressive control of blood pressure is advised, accompanied by 5 year echocardiography. Gonadoblastoma is a risk in patients with Y chromosome material,
although only about 5% of patients with Turners are thought to have cells containing some Y chromosome material. Hypothyroidism, rather than thyroid
cancer is the major risk in Turners syndrome.http://www.nhslothian.scot.nhs.uk/Services/A-
Z/DiabetesService/InformationHealthProfessionals/MUHEndocrineManagementProtocols/018_turner.pdf
Specialty: MRCP 1 May 2014
Question: 53
A 45-year-old woman presents to the Emergency Department for review. She claims to be hearing voices intermittently, which are discussing the fact that she is
worthless and she should kill herself, they are also commenting on her actions. There is a history of 3 previous drug overdoses, intermittent treatment with SSRIs for
depression, and attendance at the Emergency Department having been beaten up by her boyfriend. Physical examination is unremarkable. She is withdrawn and refuses
to engage with your questions, she does tell you however that her boyfriend is controlling her with his mobile phone, and that she can tell this by the numbers which
appear on it. Which of the following is the most likely diagnosis?
A Bipolar disease
B Depression
C Drug induced psychosis
D Personality disorder
E Schizophrenia CORRECT ANSWER
The answer is Schizophrenia - The auditory hallucinations seen here, particularly involving a commentary on the patients actions, are typical of those seen in
schizophrenia. Delusions about external control of thought also suggest schizophrenia vs the other diagnoses listed here. Whilst it is likely she is a cannabis
user, we are given no indication of significant illicit drug use. There is no indication of significant mood swings to confirm bipolar disease, and external thought
control delusions are not a feature of depression. Atypical anti-psychotics are the first line option in this situation, +/- benzodiazepines if there is significant
agitation.
Specialty: MRCP 1 May 2014
Question: 54
A 60-year-old woman with metastatic breast cancer presents with fever. She received palliative treatment with paclitaxel chemotherapy 10 days ago. She now complains
of a painful vesicular rash along her right thorax. Physical examination shows an acutely ill-looking woman. Her oral examination, as well as chest, cardiac and
abdominal examination, is normal. There is a vesicular erythematous rash on her chest. Her temperature is 39C. Laboratory examination is normal except for a
haemoglobin of 10.5 g/l, WCC 1.1 10
9
/l and platelets 30 10
9
/l. Chest X-ray is normal. Blood cultures are obtained. Which one of the following is the most
appropriate next step?
A Parenteral aciclovir and ceftazidime CORRECT ANSWER
B Granulocyte colony-stimulating factor (G-CSF)
C Platelet infusion
D Tamoxifen
E Second course of paclitaxel
Complications of chemotherapy
This patient presents with
neutropenic sepsis secondary to chemotherapy
herpes zoster infection
Management
Such patients must be immediately treated with broad-spectrum antibiotics until their neutrophil count recovers
Platelet transfusion is restricted to asymptomatic patients with a platelet count < 20 10
9
/l or thrombocytopenic patients with signs of bleeding
Role of granulocyte colony-stimulating factor
Based on the results of randomised clinical trials, therapy with granulocyte colony-stimulating factor (G-CSF) would have little effect on the course of this
patients illness
Studies to date show that the major role for G-CSF is to prevent the development of febrile neutropenia in patients who are receiving chemotherapy and who have
a greater than 40% chance of developing febrile neutropenia
Specialty: MRCP 1 May 2014
Question: 55
You are approached to take part in a new trial of an islet cell therapy derived from embryonic stem cells for Type 1 diabetes. The cells are encapsulated. Which of the
following is primary reason for encapsulating them?
A Avoid spread of neoplastic cells
B Enhance vascularisation
C Improve insulin release
D Prevent apoptosis
E Prevent autoimmune attack CORRECT ANSWER
The answer is Prevent autoimmune attack - The intention of encapsulation for stem cell derived islet cell transplants is to prevent autoimmune attack by
introducing a barrier (usually alginate) which prevents access by cells of the immune system, (primarily T lymphocytes). Cells are extensively screened and
trialled in immunosuppressed animal models of Type 1 diabetes before initiation of human studies, to exclude potential for malignant transformation. Any
barrier including alginate reduces the success of vascularisation, although the efficiency of insulin release is marginally reduced by alginate encapsulation.
Specialty: MRCP 1 May 2014
Question: 56
A 28-year-old man presents to the Emergency Department with a sudden deterioration of vision with severe blurring leading to count fingers vision only in his left eye.
Vision in the right eye remains normal and he can still perceive colours through his left eye. He is tall at 1.92m and thin, with a BMI of 21. You note that he has pectus
excavatum and a long arm span compared to the length of his body. He is hypermobile.Which of the following is the most likely cause of his visual disturbance?
A Glaucoma
B Lens dislocation CORRECT ANSWER
C Optic neuritis
D Retinal detachment
E Retinal vein occlusion
The answer is Lens dislocation - A number of features consistent with a diagnosis of Marfans syndrome are described in the scenario. As such lens dislocation
is the most likely cause of the blurred vision seen here. Retinal detachment is seen due to haemorrhage in Ehlers Danlos, not in Marfans. Acute glaucoma is
associated with pain, as is optic neuritis (optic neuritis is also associated with loss of colour vision). Retinal vein occlusion is associated with thrombophilia, not
with Marfans.
Specialty: MRCP 1 May 2014
Question: 57
A 19-year-old woman of African origin who has come to the UK to study comes to the sexual health clinic. She has been suffering from vulval warts for some time and
wants advice about the next steps in treatment, as they seem to be getting more extensive. She has no other significant health problems. Examination reveals very
significant vulval papillomata. No other abnormal findings are identified.Investigations;
Hb 10.9 g/dl
WCC 5.3 x10
9
/l
PLT 181 x10
9
/l
Na
+
137 mmol/l
K
+
4.3 mmol/l
Creatinine 100 micromol/l
ESR 62 mm/1
st
hour
CRP 52 mg/l
Which of the following would be the most appropriate next step?
A Cryotherapy
B HIV screen CORRECT ANSWER
C HPV serology
D Syphilis serology
E Topical Podophyllin
The answer is HIV screen - Extensive genital warts coupled with low grade anaemia and raised inflammatory markers raises the possibility of underlying HIV
infection. In this situation therefore HIV screening is the most appropriate next step. If HIV is identified and treatment commenced, treatment with either
cryotherapy or Podophyllin is more likely to be successful. Syphilis is associated much flatter, rather than the more papillomatous lesions seen with HPV.
Specialty: MRCP 1 May 2014
Question: 58
A 17-year-old woman is admitted to the Emergency Department complaining of lethargy, headaches, bloody diarrhoea, and most recently easy bruising over the past few
days. She is usually completely well and the only history of note is that she visited a petting farm with her younger sister a few days earlier. She takes no regular
medication. Her temperature is 38.2C, BP is 142/82 mmHg, pulse is 95/min and regular. She has bruising over her arms and legs and small petechial haemorrhages. Her
abdomen is soft, although is generally tender. Investigations;
Hb 9.8 g/dl
WCC 13.2 x10
9
/l
PLT 67 x10
9
/l
Na
+
138 mmol/l
K
+
5.3 mmol/l
Creatinine 213 micromol/l
CRP 230 mg/l
Which of the following is the most likely causative organism?
A C. jejuni
B E. coli CORRECT ANSWER
C Norovirus
D S. typhi
E S. sonnei
The answer is E coli - The clinical scenario pictured here, including a trip to a petting farm, fits with exposure to E. coli 0157. With anaemia,
thrombocytopaenia and an elevated creatinine, haemolytic uraemic syndrome seems the most likely diagnosis. A number of therapies have been trialed for
haemolytic uraemic syndrome with little evidence to support their use; these include fresh frozen plasma transfusion, heparin, urokinase, dipyridamole, Shiga
toxin-binding protein and corticosteroids. Antibiotic therapy is not thought to be of value. The other options are not associated with haemolysis.
Specialty: MRCP 1 May 2014
Question: 59
A 22 -year-old man who has spent a gap year travelling and working in the Northern Thai jungle on a school building project comes to the clinic complaining of gradual
weight loss, intermittent symptoms of irritable bowel syndrome and a chronic dry cough. He thinks he has lost 4kg over the past 6 months. Examination reveals a BP of
110/72 mmHg, pulse is 65/min and regular. Respiratory examination is normal, abdomen is soft and non-tender, his BMI is 21.Investigations;
Hb 10.9 g/dl
WCC 9.1 x10
9
/l
Eosinophils 2.3 x10
9
/l
PLT 179 x10
9
/l
Na
+
138 mmol/l
K
+
4.0 mmol/l
Creatinine 102 micromol/l
Which of the following is most likely to have precipitated his infection?
A Drinking contaminated water
B Eating contaminated food
C Mosquito bites
D Swimming in a local lake
E Walking barefoot CORRECT ANSWER
The answer is Walking barefoot - This mans symptoms are suspicious of strongyloides infection, which in chronic form may be associated with vague
symptoms of abdominal pain, features of malabsorption and eosinophilia. Strongyloides is soil-dwelling; as such walking barefoot is a significant risk for
infection. Diagnosis relies on identification of larvae in stool or duodenal fluid, or antibodies may be demonstrated in chronic infection. Swimming in a local
freshwater lake is associated with increased risk of shistasomiasis infection. Ivermectin and Albendazole are potential therapeutic options.
Specialty: MRCP 1 May 2014
Question: 60
A 19-year-old woman comes to the Gastroenterology Clinic with her brother who has severe Crohns disease and who has had multiple small bowel resections over the
past few years. She has a history of knee injuries related to running for which she takes regular non-steroidal anti-inflammatory drugs (NSAID), smokes 10 cigarettes per
day and takes the combined oral contraceptive pill.Which of the following would be considered her biggest risk factor with respect to development of Crohns in
the future?
A Cigarette smoking CORRECT ANSWER
B Family history
C Oral contraceptive pill use
D NSAID use
E Regular exercise
The answer is Cigarette smoking - Smoking is associated with a 3-4 fold increase in risk of development of Crohns disease and increases the risk of more
aggressive disease. There is a genetic component to Crohns, with 15-20% of Crohns patients having an affected relative, and 70% concordance with identical
twins. Frequent infections and use of NSAIDs may also be associated with increased risk of Crohns. The pill is associated with a very small increase in the risk
of Crohns and the risk of Crohns is reduced in patients who take regular exercise.
Specialty: MRCP 1 May 2014
Question: 61
A 35-year-old woman was admitted to the Emergency Department with a headache, neck stiffness and a purpuric rash. She was found to be suffering from
meningococcus group B. She works as a lecturer at the local university and takes classes of 10-12 students on a number of occasions each day.Which of the following is
the most appropriate intervention with respect to prophylaxis for close contacts?
A Rifampicin
B Ciprofloxacin CORRECT ANSWER
C Penicillin V
D Ceftriaxone
E No intervention needed
The answer is Ciprofloxacin - The guidance for use of prophylaxis in meningococcus is listed below, whilst her students do not require treatment, close family
contacts would:Prophylaxis indicated: Chemoprophylaxis should be offered to close contacts of cases, irrespective of vaccination status, that require public health
action (see case definitions) in the following categories: (a) Those who have had prolonged close contact with the case in a household type setting during the seven
days before onset of illness. Examples of such contacts would be those living and/or sleeping in the same household (including extended household), pupils in the
same dormitory, boy/girlfriends, or university students sharing a kitchen in a hall of residence. (b) Those who have had transient close contact with a case only if
they have been directly exposed to large particle droplets/secretions from the respiratory tract of a case around the time of admission to hospital (see section 8).In
patients who are unable to take Ciprofloxacin, Rifampicin, Ceftriaxone or Azithromycin would be potential alternatives.
http://www.hpa.org.uk/webc/hpawebfile/hpaweb_c/1194947389261
Specialty: MRCP 1 May 2014
Question: 62
A 22-year-old woman with a history of three previous Paracetamol overdoses over the past 2 years comes to the Emergency Department. She is suffering from a rash
which apparently appeared overnight. On examination it appears there is linear scarring with surrounding erythema on the flexural surfaces of both arms and behind both
knees. Routine bloods including inflammatory markers are entirely normal.Which of the following is the most likely diagnosis?
A Dermatitis artefacta CORRECT ANSWER
B Flexural eczema
C Morphea
D Psoriasis
E Pityriasis rosea
The answer is Dermatitis artefacta - The previous history of deliberate self-harm, and a linear rash which has appeared overnight is highly suggestive of
dermatitis artefacta. Management is usually shared between dermatology and psychiatry services. Management is difficult as many patients react poorly when
challenged about how their rash / skin changes came about. Flexural eczema does not follow a linear pattern, psoriasis occurs mainly on extensor surfaces and
is red and scaly in nature. Morphea is localised thickening of skin thought to be related to limited scleroderma.
Specialty: MRCP 1 May 2014
Question: 63
A 27-year-old woman comes to the Renal Clinic for review. She has a history of medullary sponge kidney, although the only sequelae of this have been hypertension,
three episodes of urinary tract infection and two presentations with renal colic over the past few years. She is 12 weeks pregnant and her GP has already switched her
anti-hypertensive medication to Methyldopa. Examination reveals a BP of 122/80 mmHg, pulse is 80/min and regular. There are no other significant
findings.Investigations;
Hb 12.5 g/dl
WCC 7.5 x10
9
/l
PLT 181 x10
9
/l
Na
+
137 mmol/l
K
+
4.3 mmol/l
Creatinine 95 micromol/l
Which of the following possible renal complications of pregnancy should she be warned about?
A Increased risk of miscarriage
B Increased risk of cyst enlargement
C Increased risk of kidney stone formation CORRECT ANSWER
D Increased risk of renal malignancy
E Increased risk of progression to end stage renal failure
The answer is Increased risk of kidney stone formation - Pregnancy in patients with medullary sponge kidney is associated with increased risk of symptomatic
renal stones, asymptomatic bacteriuria, and urinary tract infection. With appropriate management, (i.e. frequent screening of urine and antibiotic
intervention where needed), increased risk of miscarriage should not be seen. The risk of progression to end stage renal failure is very small in medullary
sponge kidney, and is not significantly increased by pregnancy.
Specialty: MRCP 1 May 2014
Question: 64
A 72-year-old woman comes to the Palliative Care Clinic for review. She has carcinoma of the breast with bony metastases. Her GP has found it difficult to control her
pain as when he increases her MST above 10mg BD she becomes excessively drowsy and nauseous. On examination her BP is 105/60 mmHg, her GCS is 15 although
she intermittently appears to fall asleep during the consultation. Her BMI is 20.Investigations;
Hb 10.5 g/dl
WCC 9.1 x10
9
/l
PLT 165 x10
9
/l
Na
+
137 mmol/l
K
+
4.3 mmol/l
Creatinine 152 micromol/l
Albumin 30 g/l
ALT 132 U/l
ALP 281 U/l
Which of the following is the most likely reason for the increased drowsiness?
A Decreased hepatic synthetic function
B Decreased renal clearance CORRECT ANSWER
C Decreased lean body mass
D Decreased total fat mass
E Increased sensitivity to morphine
The answer is Decreased renal clearance - Approximately 60% of the morphine dose is excreted in the urine after 24hrs and this patient has a significantly
elevated creatinine (152 micromol/l); as such her excretion of morphine will be significantly reduced. Fat mass impacts most on subcutaneous absorption of
morphine, such that in patients with more rapid subcutaneous absorption, a greater initial peak of morphine is therefore achieved with morphine s/c. and this
may lead to acute toxicity. Even though albumin is reduced at 30g/l, transaminases and alkaline phosphatase are only slightly elevated. The chances of hepatic
synthetic function, and morphine glucuronidation being significantly impacted are very low.
Specialty: MRCP 1 May 2014
Question: 65
A 56-year-old man presents to the Emergency Department with increasing muscle pains, lethargy and darkening of his urine over the past few days. He is taking high
dose simvastatin for a previous inferior myocardial infarction, and has been prescribed a 2 week course of Clarithromycin, (which he is half way though), for a lower
respiratory tract infection. Examination reveals a BP of 145/82 mmHg; pulse is 87/min and regular. There is pain on palpation of his upper and lower limb muscles.
Investigations;
Hb 13.1 g/dl
WCC 10.9 x10
9
/l
PLT 203 x10
9
/l
Na
+
137 mmol/l
K
+
5.8 mmol/l
Creatinine 194 micromol/l
Urea 15 U/l
CK 3230 U/l
Urine is dipstick positive to blood
What would you expect to find on urine microscopy?
A Casts
B Haemoglobin
C Myoglobin CORRECT ANSWER
D Oval bodies
E Red blood cells
The answer is Myoglobin - Clarithromycin is a potent inhibitor of CYP3A4, and as such is recognized to be a precipitant for statin related toxicity (those
metabolized primarily by 3A4 including Simvastatin and Atorvastatin). In this case rhabdomyolysis has precipitated a large rise in CK and the presence of
myoglobin, a breakdown product, in the urine. Casts may be found across a number of renal conditions:
Fatty casts / oval bodies are seen as a complication of nephrotic syndrome
Hyaline casts may be related to dehydration, exercise, or diuretics
Red blood cell casts are seen across a range of glomerulonephritides
Renal tubular epithelial cell casts are seen in renal tubular necrosis, and in kidney transplant rejection
Waxy casts are seen in chronic kidney failure
White blood cell casts are seen in urinary tract infection
Specialty: MRCP 1 May 2014
Question: 66
A 34-year-old woman presents as an urgent review following a creatinine check by her GP. He is very concerned as her creatinine has suddenly increased from 180
micromol/l to 320 micromol/l and she is feeling nauseous and lethargic. Apparently a long-term course of treatment was started for a toe infection by the GP
approximately 10 days earlier. Examination reveals a BP of 152/85 mmHg, pulse is 85/min and regular. She has an abdominal scar consistent with a renal transplant. She
has a misshapen left big toe nail.Investigations;
Hb 11.1 g/dl
WCC 4.5 x10
9
/l
PLT 191 x10
9
/l
Na
+
137 mmol/l
K
+
5.3 mmol/l
Creatinine 320 micromol/l
Which of the following co-prescribed medications is the most likely cause of the rise in creatinine?
A Azithromycin
B Cephalexin
C Co-amoxiclav
D Fluconazole CORRECT ANSWER
E Nifedipine
The answer is Fluconazole - All inhibitors of CYP3A4 and/or P-glycoprotein may lead to increased levels of Ciclosporin. Other examples include;
Ketoconazole, Fluconazole, Itraconazole and Voriconazole could more than double Ciclosporin exposure.Nicardipine, Metoclopramide, oral contraceptives,
Methylprednisolone (high dose), Allopurinol, cholic acid and derivatives, protease inhibitors, Imatinib, Colchicine, Nefazodone.Macrolide antibiotics:
Erythromycin can increase Ciclosporin exposure 4- to 7-fold, sometimes resulting in nephrotoxicity. Clarithromycin has been reported to double the exposure
of Ciclosporin. Azithromycin increases Ciclosporin levels by around 20%.Verapamil increases Ciclosporin blood concentrations 2- to 3-fold.Amiodarone
substantially increases the plasma Ciclosporin concentration concurrently with an increase in serum creatinine. This interaction can occur for a long time after
withdrawal of Amiodarone, due to its very long half-life (about 50 days).Diltiazem (at doses of 90 mg/day) can increase Ciclosporin plasma concentrations by
up to 50%.Nifedipine does not have a significant interaction potential with respect to 3A4, although co-administration with Ciclosporin increases the risk of
gingival hypertrophy. http://www.medicines.org.uk/emc/medicine/1307/SPC/Neoral+Soft+Gelatin+Capsules,+Neoral+Oral+Solution/
Specialty: MRCP 1 May 2014
Question: 67
A 74-year-old man presents to the clinic for review. He has a mobile, non tender mass in the right iliac fossa. Past medical history of note includes hypertension for
which he is treated with Amlodipine and Indapamide, and asthma for which he is treated with low dose Seretide. On examination his BP is 142/72 mmHg, pulse is
72/min and regular. He has a 4cm, firm, non tender, mobile mass in the right iliac fossa.Investigations;
Hb 10.0 g/dl
MCV 78 fl
WCC 5.9 x10
9
/l
PLT 200 x10
9
/l
Na
+
137 mmol/l
K
+
4.3 mmol/l
Creatinine 122 micromol/l
ESR 78 mm/1
st
hour
Which of the following is the most likely diagnosis?
A Caecal carcinoma CORRECT ANSWER
B Dermoid cyst
C Femoral hernia
D Inguinal hernia
E Lipoma
The answer is Caecal carcinoma - With possible occult GI blood loss, and the position of the mass, a caecal carcinoma is the most likely diagnosis. Dermoid
cysts may present in a similar way, but are extremely rare. The position as described is wrong for a femoral or inguinal hernia. A lipoma would not be
associated with probable iron deficiency anaemia. Ultrasound is non-invasive and will confirm that the mass originates from large bowel. This makes it the
initial radiological investigation of choice. It can be followed either with colonoscopy to gain a tissue diagnosis, or pre-operative staging CT scan. Caecal
carcinomas are subject only to late obstruction because faeces are predominantly liquid when they pass through the caecum.
Specialty: MRCP 1 May 2014
Question: 68
You are interested in a new gut hormone antagonist that is thought to lead to decreased hunger.Which of the following hormones is thought to stimulate hunger?
A Ghrelin CORRECT ANSWER
B GIP
C GLP-2
D Leptin
E PYY
The answer is Ghrelin - Ghrelin is secreted when the stomach is empty and drives increased hunger. Levels fall once the stomach is full and remain low for
approximately 3hrs after a meal. How quickly they begin to rise again may govern tendency to eat between meals and potentially drive increased consumption
of high calorie foods. Leptin has the opposite effect, leading to decreased appetite. GIP and PYY are incretins that drive decreased hunger, are known to
increase gut transit time, and may have central effects in reducing appetite. GLP-2 is an incretin primarily involved in driving replacement and repair of gut
mucosa cells; an agonist of GLP-2 is now marketed for the treatment of chemotherapy related GI toxicity.
Specialty: MRCP 1 May 2014
Question: 69
A 22-year-old woman presents to the Emergency Department. She has returned a few days earlier from a holiday in India and complains of abdominal bloating,
intermittent diarrhoea and foul smelling bowel gas. She has a past history of irritable bowel syndrome and takes the progesterone only pill. Examination reveals a BP of
105/70 mmHg, pulse is 70/min and regular. Her abdomen is mildly distended and she has active bowel sounds. Investigations;
Hb 12.2 g/dl
WCC 9.1 x10
9
/l
PLT 201 x10
9
/l
Na
+
x 4.3 mmol/l
K
+
4.3 mmol/l
Creatinine 103 micromol/l
Albumin 38 g/dl
ALT 35 U/l
Stool culture negative
What is the easiest way to confirm the underlying diagnosis?
A Duodenal aspiration
B Hydrogen breath test
C Repeat microscopy of fresh wet stool samples on 3 successive days
D Stool antigen test CORRECT ANSWER
E String test
The answer is Stool antigen test - In this case whilst fresh stool samples are the gold standard investigation, stool antigen testing is likely to be significantly
more convenient. The symptoms are very suggestive of giardiasis, and stool antigen testing has sensitivity and specificity both greater than 90% for giardia.
Tinidazole and Metronidazole are potential treatment options, although emergent resistance to Metronidazole (20%), is becoming a significant problem.
Aspiration of duodenal fluid and the string test, (microscopy of a string which has passed into the small bowel) are other potential alternatives, although
neither offers the convenience of stool antigen testing. Breath testing is considered for bacterial overgrowth.
Specialty: MRCP 1 May 2014
Question: 70
An 18-year-old woman with epilepsy is about to embark on a gap year trip to Africa. She has been advised to take malaria prophylaxis. She is stable from the point of
view of her epilepsy, takes regular lamotrigine and has been fit free for 2 years.Which of the following is the most appropriate choice for malaria prophylaxis?
A Doxycycline
B Malarone CORRECT ANSWER
C Mefloquine
D Chloroquine
E Proguanil
The answer is Malarone - Where there is significant chloroquine resistance, (Africa and some areas of South-East Asia), Proguanil as monotherapy is not
recommended. Chloroquine itself and Mefloquine can significantly reduce the seizure threshold. As such neither are recommended options for malaria
prophylaxis where there is a history of epilepsy. Proguanil and Atovaquone in combination (Malarone), is the recommended option. Doxycycline is an
alternative. It has efficacy in prophylaxis >90% against falciparum malaria.
Specialty: MRCP 1 May 2014
Question: 71
An 18-year-old woman is admitted to the Emergency Department following a collapse at the local swimming pool. She was attended by a first responder and paramedic
ambulance and found to be in VT with a ventricular rate of approximately 220/min. She received a DC shock on arrival at the Emergency Department. You understand
there is a history of sudden death in the family, with her father and uncle both dying at approximately 40 years of age. Her BP is now 122/72 mmHg, pulse is 65/min and
regular, there are no signs of cardiac failure.Investigations;
Hb 13.1 g/dl
WCC 8.0 x10
9
/l
PLT 201 x10
9
/l
Na
+
137 mmol/l
K
+
4.0 mmol/l
Creatinine 100 micromol/l
ECG QT interval 510ms
Where is the defect most likely to be found?
A Calcium channel
B Chloride channel
C Magnesium channel
D Potassium channel CORRECT ANSWER
E Sodium channel
The answer is Potassium channel - The most likely underlying diagnosis is congenital long QT syndrome. In this case the mode of inheritance is most likely to
be autosomal dominant, although sporadic and recessive forms are also described. A number of defects in potassium, calcium and sodium channels have been
described, although genetic mutations leading to potassium channel defects are most common:
Type of LQTs Chromosomal locus Mutated gene Ion Current affected
LQT1 11p15.5 KVLQT1 or KCNQ1 (heterozygotes) Potassium (I
ks
)
LQT2 7q35-36 HERG, KCNH2 Potassium (I
kr
)
LQT3 3p21-24 SCN5A Sodium (I
Na
)
LQT4 4q25-27 ANK2, ANKB Sodium, potassium and calcium
LQT5 21q22.1-22.2 KCNE1 (heterozygotes) Potassium (I
ks
)
LQT6 21q22.1-22.2 MiRP1, KNCE2 Potassium (I
kr
)
LQT7 (Anderson syndrome) 17q23.1-q24.2 KCNJ2 Potassium (I
kr
)
LQT8 (Timothy syndrome) 12q13.3 CACNA1C Calcium (I
Ca-Lalpha
)
LQT9 3p25.3 CAV3 Sodium (I
Na
)
LQT10 11q23.3 SCN4B Sodium (I
Na
)
LQT11 7q21-q22 AKAP9 Potassium (I
ks
)
LQT12 SNTAI Sodium (I
Na
)
JLN1 11p15.5 KVLQT1 or KCNQ1 (homozygotes) Potassium (I
ks
)
JLN2 21q22.1-22.2 KCNE1 (homozygotes) Potassium (I
ks
)
Beta-blockers are the initial therapy of choice, which prevent further episodes of VT in approximately 70% by reducing sympathetic outflow. In those who fail
to respond, stellate ganglionectomy is an option, and there is potential for implantable cardioverter defibrillator (ICD) implantation. In those who have
suffered a cardiac arrest, ICD implantation may be considered initial therapy of choice.
Specialty: MRCP 1 May 2014
Question: 72
A 62-year-old patient is being managed on the CCU 4 hrs post percutaneous coronary intervention (PCI) for anterior myocardial infarction. He is well post procedure
and is not in cardiac failure. His BP is maintained at 122/72 mmHg. You are asked to see him by the nursing staff as they have noticed he has developed an LBBB
pattern on his ECG with a ventricular rate of 108/min (regular). Potassium and magnesium are in the normal range.Which of the following is the most appropriate
intervention?
A DC cardioversion
B IV Amiodarone
C IV Magnesium
D No action needed CORRECT ANSWER
E Oral Sotalol
The answer is No action needed - Development of LBBB is unsurprising post anterior MI, this patient has preserved BP and is not in cardiac failure. His pulse
rate is likely to settle spontaneously, and in the absence of chest pain, no intervention is required. In VT with preserved BP, loading with Amiodarone may be
considered. If there is VT with any suggestion of haemodynamic instability, then DC cardioversion is advised. IV Magnesium is of most value in the treatment
of torsades, even when magnesium levels are in the normal range.
Specialty: MRCP 1 May 2014
Question: 73
A 36-year-old woman has been investigated for hypercalcaemia and is found to have hyperparathyroidism. There is a family history that makes an underlying diagnosis
of multiple endocrine neoplasia type 1 (MEN 1) suspected. Which one of the following best fits with the clinical features of this condition?
A It is inherited in an autosomal recessive manner
B It includes medullary-cell carcinoma of the thyroid
C It includes phaeochromocytoma
D Primary hyperparathyroidism is common CORRECT ANSWER
E Patients have a normal life expectancy
Multiple endocrine neoplasia
Multiple endocrine neoplasia (MEN) is an autosomal dominant disorder characterised by a high frequency of peptic ulcer disease (ZollingerEllison syndrome)
and primary endocrine abnormalities involving the pituitary, parathyroid and pancreas
Medullary-cell carcinoma of the thyroid occurs in MEN 2a and MEN 2b
Phaeochromocytomas occur in MEN 2a and MEN 2b
Primary hyperparathyroidism
Primary hyperparathyroidism is the most common abnormality
Parathyroid hyperplasia occurs in MEN 1 and MEN 2a
Prognosis
Patients have a decreased life expectancy, with a 50% probability of death by the age of 50 years
Specialty: MRCP 1 May 2014
Question: 74
A 63-year-old woman who is treated with long-term haemodialysis for renal failure related to Type 1 diabetes comes to the clinic for review. She complains of persistent
tiredness and lethargy. She also has extreme thirst which means it is very hard for her to stick to her fluid intake limit, and generalised bony aches and pains. She takes a
number of medications including vitamin D, calcium supplements and a phosphate binder. She is pale and thin with a BMI of 21. Her BP is 155/85
mmHg.Investigations;
Hb 10.1 g/dl
WCC 7.4 x10
9
/l
PLT 192 x10
9
/l
Na
+
137 mmol/l
K
+
5.0 mmol/l
Creatinine 697 micromol/l
PO
4
3-
1.8 mmol/l
Ca
++
2.82 mmol/l
PTH 7.5 pmol/l
Which of the following is the most likely diagnosis?
A Hypervitaminosis D
B Pseudohypoparathyroidism
C Primary hyperparathyroidism
D Secondary hyperparathyroidism
E Tertiary hyperparathyroidism CORRECT ANSWER
The answer is Tertiary hyperparathyroidism - The high PTH in the presence of hypercalcaemia is indicative of tertiary hyperparathyroidism. It fits with the
history of end stage renal failure. PTH initially rises in CRF in response to low levels of serum calcium because of disordered vitamin D metabolism. Calcium
and vitamin D supplements are prescribed in an attempt to suppress PTH, coupled with phosphate binders in the event that phosphate begins to rise.
Eventually however, as here, PTH can escape its negative feedback loop with respect to calcium, leading to hypercalcaemia and elevated PTH.
Hypervitaminosis D would be associated with suppressed PTH.
Specialty: MRCP 1 May 2014
Question: 75
A 23-year-old woman is brought to the Emergency Department from the local airport for review. Apparently some 20 minutes into a flight from Spain she had an
episode of loss of consciousness (LoC), lasting some 40 seconds, whilst strapped into her seat. According to a relative she had been drinking beer before boarding the
flight and complained of being hot just before passing out. There was some twitching of both arms at the time of loss of consciousness. She quickly regained normal
function after being laid flat in the aisle. There is no past medical history of note. Clinical examination, including full neurological exam is entirely
normal.Investigations;
Hb 12.9 g/dl
WCC 7.9 x10
9
/l
PLT 201 x10
9
/l
Na
+
137 mmol/l
K
+
4.3 mmol/l
Creatinine 85 micromol/l
CT head unremarkable
12 lead ECG normal
Which of the following is the most likely diagnosis?
A Alcohol withdrawal seizure
B Cardiac syncope
C Complex partial seizure
D Absence epilepsy
E Vasovagal episode CORRECT ANSWER
The answer is Vasovagal episode - The most likely explanation is that this woman suffered a faint whilst strapped into her plane seat, was unable to attain a
horizontal position and this lead to the twitching and the short period of loss of consciousness. The rapid recovery and lack of incontinence count against a
diagnosis of epilepsy. Alcohol withdrawal seizures are only seen after a more prolonged period of abstinence in a physically dependent alcoholic, and the lack of
a significant previous medical history makes cardiac syncope less likely.
Specialty: MRCP 1 May 2014
Question: 76
The test performance of a new screening test for breast cancer in 100 subjects is illustrated below:
Disease confirmed Free of disease
Test outcome positive 65 5
Test outcome negative 15 15
What is the sensitivity of the test?
A 15%
B 33%
C 50%
D 81% CORRECT ANSWER
E 93%
The answer is 81% - Sensitivity = True positives / (true positives + false negatives) which equals 65/(65+15) = 81%The positive predictive value (PPV) is also
important in this type of test, to avoid significant patient anxiety when no underlying carcinoma is present. The PPV calculates to be 65 /(65+5) = 93%. The
negative predictive value for this test is 15/(15+15), which equals 50%. This is clearly of significant concern because a number of cases may be missed if this test
alone was used as the screen for breast cancer.
Specialty: MRCP 1 May 2014
Question: 77
A 67-year-old patient is referred to the Renal Clinic for review. His GP has noted a deteriorating creatinine from 136 to 210 micromol/l and a corresponding fall in his
GFR. He has had Type 2 diabetes for the past 7 years for which he takes Gliclazide 160mg BD, Aspirin 75mg and Simvastatin 20mg. He is also prescribed Amlodipine
5mg daily for treatment of hypertension. 24hr mean systolic BP is 136mmHg.Investigations;
Hb 10.9 g/dl
WCC 6.2 x10
9
/l
PLT 189 x10
9
/l
Na
+
137 mmol/l
K
+
4.9 mmol/l
Creatinine 210 micromol/l
Glucose 8.1 mmol/l
HbA1c 54 mmol/mol (7.1%)
Urine albustix positive
Which change would you make to the anti-hypertensive medication?
A Add Bendroflumethiazide
B Add Bisoprolol
C Add Diltiazem
D Add Ramipril CORRECT ANSWER
E Increase Amlodipine
The answer is Add Ramipril - This patient in all likelihood has diabetic nephropathy, and despite the fact his creatinine is significantly elevated, he should still
be offered a trial of ACE inhibitor. Regular monitoring of renal function post ACE inhibition is important as the rise in creatinine may be unrelated to diabetes
and in fact be due to renovascular disease. In patients who cannot tolerate ACE inhibition, evidence has shown that Diltiazem may reduce proteinuria in
patients with diabetic nephropathy. If hypertension persists post initiation of ACE inhibitor, a thiazide like diuretic such as Indapamide would be the next
appropriate step.
Specialty: MRCP 1 May 2014
Question: 78
A 34-year-old man with a history of ulcerative colitis comes to the Emergency Department for review. He was started on high dose oral corticosteroids in the
Gastroenterology Clinic 3 days earlier because he was opening his bowels 6-10 times / day with bloody diarrhoea. His symptoms have not improved; if anything his
abdomen is more distended and there has been no difference in his bowel frequency. Examination reveals a BP of 105/70 mmHg, his pulse is 95/min and regular,
temperature is elevated at 38.2C. His abdomen is distended and diffusely tender, he has active bowel sounds. Investigations;
Hb 10.9 g/dl
WCC 13.2 x10
9
/l
PLT 550 x10
9
/l
Na
+
137 mmol/l
K
+
3.3 mmol/l
Creatinine 132 micromol/l
Urea 10.2 mmol/l
ESR 85 mm/1
st
hour
Which of the following is the most important next investigation?
A Abdominal X-ray CORRECT ANSWER
B CT abdomen
C Faecal calprotectin
D Sigmoidoscopy
E Stool culture
The answer is Abdominal x-ray - The key step in a patient with active colitis not responding to steroids, with worsening abdominal distension is to rule out
toxic megacolon with an abdominal x-ray. If it is ruled out then there is the opportunity to step up medical therapy with IV corticosteroids and Ciclosporin.
Sigmoidoscopy should be considered to obtain biopsy specimens to confirm active inflammation, and faecal calprotectin is a marker of active inflammation.
Stool culture is important to rule out coexistent bacterial infection.
Specialty: MRCP 1 May 2014
Question: 79
A 78-year-old woman is brought into the Emergency Department by her daughter. Her daughter is very concerned, as her mother has developed nausea, vomiting,
pyrexia and confusion after spending the day sitting out in the sun with her family. On examination she is pyrexial 38.0C, her BP is 110/72 mmHg, pulse is 88/min and
regular and she looks dehydrated.Which of the following physiological changes is likely to have increased her risk of heat stroke?
A Decreased peripheral dilatation
B Decreased sweating CORRECT ANSWER
C Increased basal metabolic rate
D Increased sweating
E Increased peripheral vasodilatation
The answer is Decreased sweating - A number of physiological changes occur in the elderly, but the one that is most associated with increased risk of heat
stroke is a loss of ability to sweat. Increasing age is associated with increased arteriolar calcification, which may reduce ability to lose heat by peripheral
vasodilation, although this is less of a contributor to reduced heat loss than decreased sweating. Consumption of alcohol in the elderly population is also
thought to contribute to risk of both hypo and hyperthermia. Clin Geriatr Med. 1993 Aug;9(3):621-39
Specialty: MRCP 1 May 2014
Question: 80
A 19-year-old student is brought to the Emergency Department by paramedic ambulance. According to her friends she has been complaining of a headache and sore
throat over the past few days, and when they tried to wake her in the morning she had become drowsy and confused. Examination reveals a pyrexia 38.5C, her pulse is
100/min and regular and her BP is 85/min and regular. She has skin mottling with a maculopapular rash and areas of petechial haemorrhage. Her GCS is 13 and she has
obvious signs of meningism. You are unable to see her fundi.Which of the following is the most appropriate next step?
A IV Ceftriaxone CORRECT ANSWER
B IV Clarithromycin
C IV Co-amoxiclav
D IM Penicillin
E IV Penicillin
The answer is IV Ceftriaxone - There is a very high suspicion of N. meningitidis infection here, and guidelines recommend commencing IV Ceftriaxone
without delay. Vancomycin is considered as additional therapy where there is a history of overseas travel or exposure to multiple antibiotics. Corticosteroids
may reduce hearing loss and neurological sequelae although they do not impact on overall mortality. Circulatory support, in the first instance with IV fluids is
also required. Pre-hospital Penicillin or Cefotaxime are recommended, although they should not obviously delay transport to the hospital.
Specialty: MRCP 1 May 2014
Question: 81
A 61-year-old woman presents to the clinic for review. She has hyperpigmented, thickened skin in axilla and groin regions. She reports that she often feels full after a
meal, and has lost 8kg in weight over the past 6 months although she did not intend this to happen. There is a history of hypertension and mild asthma but nil else of note.
Examination reveals darkened, thickened areas of skin affecting her axillae and both sides of the groin. Her BMI is 29. There are no other findings on clinical
examination.Which of the following is a possible underlying malignancy?
A Bronchial carcinoid
B Gastric adenocarcinoma CORRECT ANSWER
C Hepatocellular carcinoma
D Hodgkins disease
E Pancreatic carcinoma
The answer is Gastric adenocarcinoma - Whilst also being associated with diabetes mellitus and insulin resistance syndromes, acanthosis nigricans, the
diagnosis here, is also associated with gastric cancer. The underlying diagnosis is supported by the unintentional weight loss over the past few months.
Bronchial carcinoid is associated with facial flushing, but only usually after metastases have occurred. Necrolytic migratory erythema is associated with
pancreatic tumours. Hepatocellular carcinoma is associated with pityriasis rotunda. Hodgkins disease is associated with acrodermatosis paraneoplastica.
Specialty: MRCP 1 May 2014
Question: 82
A 71-year-old man who is treated with long-term peritoneal dialysis presents to the Emergency Department with dull abdominal pain and inability to drain out properly.
He says that his overnight bag has become cloudy and he has been suffering from a fever. He has opened his bowels that morning and suffered from slight diarrhoea. On
examination his temperature is 38.0C; BP is 110/60 mmHg, pulse is 85/min and regular. His abdomen is soft, although is generally tender, and it is clear there is
residual peritoneal dialysis fluid present. Investigations;
Hb 10.1 g/dl
WCC 12.3 x10
9
/l
PLT 198 x10
9
/l
Na
+
137 mmol/l
K
+
4.9 mmol/l
Creatinine 540 micromol/l
Glucose 8.1 mmol/l
CRP 189 mg/l
A peritoneal tap is plannedWhich of the following would meet one of the diagnostic criteria for spontaneous bacterial peritonitis?
A Albumin 22g/l
B Glucose 8.1 mmol/l
C LDH 200 U/l
D Neutrophil count 265 neutrophils per mm
3
CORRECT ANSWER
E Protein 1.2 g/l
The answer is Neutrophil count 265 neutrophils per mm
3
- Spontaneous bacterial peritonitis is defined by the presence of >250 polymorphonuclear cells
(PMN)/ mm
3
in ascites, in the absence of an intra-abdominal source of infection or malignancy. The presence of glucose is to be expected within the dialysis
fluid, because the largest component of the majority of dialysis fluids is glucose. Protein and LDH do not form part of the diagnostic criteria for spontaneous
bacterial peritonitis. IV Gentamicin and oral fluoroquinolones are both potential treatment options.http://gut.bmj.com/content/61/2/297.full#sec-20
Specialty: MRCP 1 May 2014
Question: 83
A 45-year-old woman undergoing her third cycle of intensive chemotherapy for breast cancer is admitted via the medical take. She has been suffering from a sore throat
for some 1216 h and has now begun a fever with profuse sweating. On admission to the Emergency Department she has a pyrexia of 38.2C. She has an indwelling,
tunnelled, subcutaneous catheter for chemotherapy. Which one of the following micro-organisms is most likely to be responsible for her fever?
A Candida albicans
B Acinetobacter sp
C Escherichia coli
D Klebsiella sp
E Staphylococcus epidermidis CORRECT ANSWER
Febrile neutropenia Aetiology
Commonly, febrile neutropenia is said to be due to Gram-positive organisms such as Staphylococcus epidermidis or Streptococcus viridans (around 60% of cases)
Around 30% of cases may be due to Gram-negative bacteria such as E. coli,Klebsiella spp or Pseudomonas aeruginosa
Other organisms, such as candida, corynebacterium, cytomegalovirus(CMV) or varicella zoster virus, make up the other 10% of cases
A microbiological diagnosis, however, is only made in around 40% of cases
Source of infection
Indwelling lines are often the cause of Staph. epidermidis infection
Infection with viridans streptococci may be related to mucositis or previous quinolone treatment
It is important to take multiple samples of blood from indwelling lines and peripheral sites for culture, and to culture urine and faeces, arrange viral serology and to
send the tips of any lines for culture
Treatment
Antibiotic therapy should be started before the results of the cultures are available, but, if possible, not before samples are taken
Standard regimes include ceftazidime and vancomycin; piperacillin and gentamicin as second-line choice; and imipenem/cilastatin as third-line choice
Platelet counts may fall rapidly in response to sepsis, and platelet transfusion may be necessary in some cases
Specialty: MRCP 1 May 2014
Question: 84
A 72-year-old man presents with progressively worsening generalised erythema with severe itching, lethargy and night sweats over the course of the past 7 months. He
has also lost 5kg in weight. Past medical history includes diabetes mellitus, a previous inferior myocardial infarction and a left total hip replacement, but nil else of note.
On examination his BP is 138/78 mmHg, pulse is 70/min and regular. There is generalised exfoliative dermatitis. A punch skin biopsy confirms extensive invasion by
atypical T- cells. His bloods show anaemia with a marked elevation in ESR.Which of the following is the most likely diagnosis?
A Cutaneous T cell lymphoma CORRECT ANSWER
B Plaque psoriasis
C Pemphigus vulgaris
D Allergic contact dermatitis
E Lichen planus
The answer is Cutaneous T cell lymphoma The generalised erythema, coupled with this patients age, symptoms of a significant systemic illness and extensive
atypical T-cell invasion on biopsy, fits best with cutaneous T-cell lymphoma. Systemic treatment, such as oral retinoids, recombinant interferon-alpha, fusion
toxins, monoclonal antibodies, and single-agent chemotherapy, can be used sequentially to palliate symptoms from more advanced cutaneous T-cell lymphoma
as here, where the large size of the area to be treated effectively rules out topical therapies.Pemphigus is associated with a blistering rash, and the typical skin
changes associated with lichen planus are intensely itchy 2-5 mm red or violet shiny flat-topped papules with white streaks known as Wickham's striae.
Specialty: MRCP 1 May 2014
Question: 85
A 65-year-old former secretary comes to the Rheumatology Clinic for review. She has found it increasingly difficult to knit because of pain and stiffness affecting the
fingers of both hands. She also has pain in both hips and knees but puts this down to old age. Other past history of note includes hypertension and Type 2 diabetes.
Examination reveals a BP of 155/85 mmHg, pulse is 75/min and regular. Her BMI is 32. There are bony nodules over the proximal and distal interphalangeal joints, and
she appears to have limited external rotation of both hips and pain on knee flexion.Investigations;
Hb 13.1 g/dl
WCC 9.0 x10
9
/l
PLT 198 x10
9
/l
Na
+
137 mmol/l
K
+
4.3 mmol/l
Creatinine 102 micromol/l
CRP 12 mg/l
RhF negative
ANA negative
Glucose 8.1 mmol/l
HbA1c 67 mmol/mol (8.3%)
Which of the following is the most likely diagnosis?
A Gouty arthritis
B Osteoarthritis CORRECT ANSWER
C Pseudogout
D Rheumatoid arthritis
E Seronegative arthritis
The answer is Osteoarthritis - This patient has typical changes in her hands occurring as a result of occupational osteoarthritis related to her previous job as a
secretary. The changes in her hips and knees are also suggestive of osteoarthritis, albeit related to her obesity. The normal inflammatory markers and negative
RhF and ANA count against an underlying inflammatory process. Gout and pseudogout are more typically oligoarthritides affecting either the knee or the
great toe. Simple analgesia is the intervention of choice; she should also be encouraged to reduce her weight so that deterioration of her hips and knees is
slowed.
Specialty: MRCP 1 May 2014
Question: 86
A 29-year-old nurse collapses with symptoms of anaphylaxis whilst eating a home prepared fruit salad in the hospital canteen. She is known to have latex allergy and is
careful to use nitrile gloves at all times in her work at the hospital. On admission to the Emergency Department her BP is 100/60 mmHg, pulse is 110/min. She has facial
flushing and significant stridor and requires s/c adrenalin, fluid loading and IV hydrocortisone.Which of the following fruits is likely to have been found in the fruit
salad?
A Apple
B Blueberry
C Grapefruit
D Melon CORRECT ANSWER
E Strawberry
The answer is Melon - The history here is suggestive of latex-fruit syndrome, where proteins contained within certain fruits have cross-reactivity with
antibodies to latex antigens. Reactions to avocado, banana, kiwi fruit, melon and chestnut have all been reported. Other reactions to peach and tomato have
also been reported, although these may just be representative of patients with multiple food allergies. Where a trigger food is recognised, this should of course
be avoided in the future.
Specialty: MRCP 1 May 2014
Question: 87
A 23-year-old woman presents to the sexual health clinic with a number of flat, macular lesions in the region of her vulva. She admits to having unprotected sexual
intercourse on two occasions during a holiday a few months earlier, following which she had a painless spot in her vulval region, which took some time to heal. There is
no past medical history of note, and her only medication is the oral contraceptive pill. Examination reveals a number of flattened erythematous macular lesions over the
vulval area. There is evidence of inguinal lymphadenopathy.Which of the following would be the most useful next investigation in this situation?
A Antinuclear antibodies
B HIV serology
C HPV serology
D Skin biopsy
E Syphilis serology CORRECT ANSWER
The answer is Syphilis serology - The description of the lesions seen here is suspicious for condyloma lata, consistent with secondary syphilis. As such serology
testing is indicated followed by appropriate antibiotic therapy; Penicillin, Azithromycin and Doxycycline would all be potential options. A full sexual health
screen including HIV serology is of course indicated, but would not be immediately informative in identifying the appropriate intervention for the rash seen
here. As the rash is not consistent with condyloma accuminata, HPV serology is likely to be uninformative.
Specialty: MRCP 1 May 2014
Question: 88
A 42-year-old woman presents to the Endocrine Clinic for review. She has hypertension and Type 2 diabetes and has become very concerned, as her fingers have started
to swell such that her rings no longer fit her. She takes a number of medications for control of blood pressure and diabetes mellitus. Examination reveals a BP of 155/95
mmHg, pulse is 75/min and regular. She has thickened hyperpigmented skin under both arms. There is also a prominent lower jaw and her fingers appear swollen. She is
perspiring at rest. Her BMI is 27.Investigations;
Hb 13.1 g/dl
WCC 7.1 x10
9
/l
PLT 200 x10
9
/l
Na
+
137 mmol/l
K
+
4.3 mmol/l
Creatinine 115 micromol/l
Prolactin 800 mU/l
TSH 2.4 U/l
Glucose 9.1 mmol/l
Which of the following is the most useful next investigation?
A Glucagon
B IGF-1 CORRECT ANSWER
C MRI pituitary
D 24hr urinary cortisol
E 24hr urinary catecholamines
The answer is IGF-1 - The prominent lower jaw, soft tissue oedema, acanthosis nigricans, hypertension and diabetes are all pointers to an underlying diagnosis
of acromegaly. As such IGF-1 as a non-invasive investigation is the most appropriate next step. MRI can then be considered to confirm the presence of a
growth hormone producing adenoma. The prolactin is elevated, but at 800 mU/l this is most likely to be due to local pressure effect rather than to a prolactin-
secreting tumour. The body habitus is more suggestive of acromegaly than Cushings disease (a potential alternative diagnosis here).
Specialty: MRCP 1 May 2014
Question: 89
A 17-year-old man with a history of cystic fibrosis comes to the Respiratory Clinic for review. Despite pancrelipase tablets and eating a high calorie high carbohydrate
diet, he is finding it very difficult to maintain his weight. He asks about using vitamin supplements. Which of the following vitamins is it most important to
supplement in cystic fibrosis?
A Vitamin A CORRECT ANSWER
B Vitamin B6
C Vitamin B12
D Vitamin C
E Folate
The answer is Vitamin A - Cystic fibrosis results in malabsorption of fat, therefore vitamin supplements for fat-soluble vitamins are recommended. These
include vitamins A, D, E and K. Recommended daily levels for vitamin supplementation are: Vitamin A 4,000 -10,000 IU (1200-3,000 mcg), vitamin D 800-
2,000 IU (20-50mcg) and vitamin E 150-300 IU (100-200 mg). Studies of B vitamins in patients with cystic fibrosis do not suggest significant B vitamin
deficiency.
Specialty: MRCP 1 May 2014
Question: 90
A 28-year-old man is admitted to the Emergency Department 1hr after the onset of fast palpitations. He has drunk 6 pints of beer during the course of the evening
having completed a half marathon the previous afternoon. There is a history of one previous episode of palpitations 1 year earlier, again after a period of increased
alcohol consumption. He is usually fit and well, works as a solicitor and runs 30-40km per week. Examination reveals a BP of 110/80 mmHg, a pulse of 180/min,
irregular. He is not in cardiac failure.Investigations;
Hb 13.1 g/dl
WCC 7.1 x10
9
/l
PLT 191 x10
9
/l
Na
+
137 mmol/l
K
+
4.5 mmol/l
Creatinine 105 micromol/l
Glucose 5.1 mmol/l
CXR normal heart size, no signs of cardiac failure
ECG fast AF, no signs of ischaemia
Which of the following is the most appropriate intervention?
A Amiodarone
B Bisoprolol
C Digoxin
D Flecainide CORRECT ANSWER
E Verapamil
The answer is Flecainide - This patient has an episode of paroxysmal AF with no previous history of cardiovascular disease. It is likely the episode has been
precipitated by excessive alcohol consumption. Flecainide is associated with a greater chance of attaining sinus rhythm than Amiodarone and is therefore the
intervention of choice. Amiodarone is the preferred option to attain sinus rhythm in patients with a history of structural or ischaemic heart disease. Bisoprolol
is the preferred oral option for maintenance of sinus rhythm in patients with a history of AF. Digoxin and Verapamil are options for slowing the ventricular
rate but are not effective in attaining sinus rhythm. http://www.nice.org.uk/Guidance/CG180
Specialty: MRCP 1 May 2014
Question: 91
A 54-year-old man who has moved to the UK from Romania has lost 4kg in weight over the past 4 months. He has a chronic cough and smokes 20 cigarettes per day.
Examination reveals a BP of 122/72 mmHg, pulse is 80/min and regular, he is apyrexial. There is no significant lymphadenopathy. Auscultation of the chest reveals
bilateral wheeze. Investigations;
Hb 13.1 g/dl
WCC 11.2 x10
9
/l
PLT 203 x10
9
/l
Na
+
135 mmol/l
K
+
4.3 mmol/l
Creatinine 110 micromol/l
ESR 92 mm/1
st
hour
CXR Evidence of right lateral pleural thickening
Quantiferon gamma test positive
Which of the following is the most appropriate way to confirm a diagnosis of pleural tuberculosis?
A Bronchio-alveolar lavage
B Mycobacterium tuberculosis PCR
C Pleural aspirate microscopy and culture
D Pleural biopsy microscopy and culture CORRECT ANSWER
E Sputum microscopy and culture
The answer is Pleural biopsy microscopy and culture - Mycobacterium tuberculosis PCR may be positive, although it would not localise the site of tuberculosis
infection. The yields from broncho-alveolar lavage (BAL), pleural aspiration and sputum microscopy are all much lower than guided pleural biopsy. Pleural
biopsy should take place under thoracoscopic or radiological guidance to ensure that the abnormal area seen on CXR is biopsied, since the yield from blind
biopsy is much lower. Microscopy of a smear may yield mycobacteria, or they may only be seen following culture.
Specialty: MRCP 1 May 2014
Question: 92
A 51-year-old woman comes to the Endocrine Clinic for review. She has not had a menstrual period for the past 6 months, and is suffering from episodes of facial
flushing, two urinary tract infections over the past few months, and pain on intercourse. She has no past medical history of note. Her BPis 115/83 mmHg, pulse is 65/min
and regular. Her BMI is 22. Her FSH is measured at >40 U/l. She asks about use of HRT. Which of the following is the main indication for HRT?
A Reduction of breast cancer
B Reduction of cardiovascular risk
C Reduction of menopausal symptoms CORRECT ANSWER
D Reduction of ovarian cancer
E Reduction of venous thromboembolism
The answer is Reduction of menopausal symptoms - HRT is only really indicated now for reduction of menopausal symptoms, with maximal effect on
vasomotor symptoms seen within three months. It has positive effects on symptoms of vaginal atrophy, although similar benefit can be gained from local
therapies including oestrogen cream and non-hormonal lubricants. Where possible time on HRT should be minimised because it is associated with increased
risk of:
Coronary artery disease and stroke (slight increase in risk)
Endometrial cancer (2-12 fold in patients using an oestrogen-only preparation)
Breast cancer (risk apparent after 3 years of use, returns to baseline within 5 years of stopping therapy)
Venous thromboembolism (1.3 to 3 fold increase in risk)
https://www.medicines.org.uk/emc/medicine/1363#CLINICAL_PRECAUTIONS
Specialty: MRCP 1 May 2014
Question: 93
A 77-year-old man presents to the Cardiology Clinic for review. He has been referred by his GP because of a murmur suspicious of aortic stenosis (ejection systolic,
loudest at the left sternal edge), and difficult to control hypertension. His BP is measured by the GP at 132/100 mmHg. Which of the following heart sounds would be
most indicative of severe aortic stenosis?
A Loud S1
B Loud S2
C Loud S4
D Quiet S1
E Quiet S2 CORRECT ANSWER
The answer is Quiet S2 - A2, is usually diminished or absent in significant aortic stenosis due to the calcified, immobile aortic value. The presence of a normal
or accentuated A2 speaks against the presence of severe aortic stenosis. S1 is usually normal or soft, but changes in the loudness of S1 are not particularly
indicative of severe AS. Prominent S4 is due to forceful atrial contraction into a hypertrophied left ventricle, whilst it is indicative of severe disease in younger
patients, it is less significant with respect to severity of disease in the older population.
Specialty: MRCP 1 May 2014
Question: 94
An 18-year-old woman presents to the Emergency Department with facial and lower limb oedema. She believes she had a similar episode as a child that resolved with a
course of steroids and has had no significant problems since then. Her BP is 122/82 mmHg, pulse is 78/min and regular. There is pitting oedema affecting both lower
limbs.Investigations;
Hb 12.9 g/dl
WCC 10.0 x10
9
/l
PLT 201 x10
9
/l
Na
+
137 mmol/l
K
+
5.2 mmol/l
Creatinine 95 micromol/l
Albumin 25 g/l
Urine protein +++
Renal biopsy shows minimal change disease
Which of the following is the most appropriate intervention?
A Cyclophosphamide
B Lisinopril
C Conservative management
D Prednisolone CORRECT ANSWER
E Valsartan
The answer is Prednisolone - Minimal change disease usually responds to high dose steroids within 6 weeks of initiating therapy. In those where the response is
inadequate, steroids can be considered for longer than 6 weeks and a steroid-sparing agent such as Ciclosporin added. In those where proteinuria is marked,
or who become hypertensive, an ACE inhibitor or angiotensin II receptor blocker (ARB) should be considered in an attempt to reduce progression of renal
disease. Chronic renal failure post-minimal change disease is extremely rare, although recurrent episodes may be seen, particularly in children.
Specialty: MRCP 1 May 2014
Question: 95
A 42-year-old woman who has undergone a kidney transplant for diabetes related end-stage renal failure comes to the clinic complaining of excessive hairiness. She
takes a number of medications including anti-hypertensives, immunosuppressants and corticosteroids. Examination confirms significant growth of body and facial
hair.Which of the following is the most likely cause?
A Azathioprine
B Ciclosporin CORRECT ANSWER
C Nifedipine
D Prednisolone
E Ramipril
The answer is Ciclosporin - Ciclosporin is well recognised as a cause of hirsuitism, which is listed as very common on the Ciclosporin summary of product
characteristics. Other pharmaceutical causes of hirsuitism include anabolic steroids, Danazol, Minoxidil, Metoclopramide, Methyldopa, Phenothiazines and
Progestogens. Nifedipine leads to gum hypertrophy and prednisolone is a cause of weight gain and acne, but neither significantly increase the risk of
hirsuitism. The main risk of Azathioprine is the potential for drug interaction with Allopurinol, leading to inhibition of metabolism of 6-mercaptopurine and
hence risk of bone marrow suppression.
Specialty: MRCP 1 May 2014
Question: 96
A 52-year-old man comes to the Rheumatology Clinic for review. He complains of pain over the medial aspect of his elbow, radiating into the forearm, aggravated by
pronation and flexion of the wrist. There is no past medical history of note; he tells you however that he has recently taken up golf. On examination pain is localised over
the insertion of the wrist flexors on the medial side of the elbow. Which of the following is the most likely diagnosis?
A Brachioradialis tendonitis
B De Quervains tenosynovitis
C Lateral epicondylitis
D Medial epicondylitis CORRECT ANSWER
E Radiocapitellar arthritis
The answer is Medial epicondylitis - The clue here is the taking up of golf as a hobby, which is well recognised as a cause of medial epicondylitis. The point
tenderness over the medial epicondyle and pain on pronation and flexion of the wrist confirms the diagnosis. Treatment with NSAIDS and physiotherapy is the
mainstay of therapy, and he should of course be advised to stop golf for a while. Medial tendon release may be of value in patients who fail to respond to
conservative management. Lateral epicondylitis is known as tennis elbow and is associated with pain over the lateral epicondyle. Radiocapitellar arthritis is
associated with pain over the radial head, de Quervains with pain at the base of the thumb, and brachioradialis tendonitis with pain over the upper lateral
supracondylar ridge of the humerus and the lateral intermuscular septum.
Specialty: MRCP 1 May 2014
Question: 97
A 20-year-old man is admitted for radiofrequency ablation for recurrent paroxysmal atrial fibrillation. He has failed trials of both oral Flecainide and then later Sotalol.
On examination in the hospital, his BP is 110/70 mmHg, pulse is 65/min and regular. 12 lead ECG is unremarkable.Which of the following is the commonest site for
radiofrequency ablation?
A SA node
B Superior vena cava
C Mitral valve
D AV node
E Pulmonary veins CORRECT ANSWER
The answer is Pulmonary veins - With regards to AF, radiofrequency ablation has a success rate approaching 85% when wide circumferential ablation is
performed around the areas where pulmonary veins join with the atrium. The goal is to electrically isolate rapid electrical activity arising from inside the
veins, or adjacent to the pulmonary vein ostia, from the rest of the left atrium. AV nodal radiofrequency ablation is performed less often because it is
associated with increased risk of pacemaker requirement. AV nodal re-entrant tachycardia can be treated with ablation at the level of the coronary sinus os.
Specialty: MRCP 1 May 2014
Question: 98
A 17-year-old woman comes to the Endocrine Clinic for review. She had three or four periods at the age of 13, and was then started on the oral contraceptive pill
because of severe menstrual pain and heavy bleeding. She is worried, as she has had no periods for the past 9 months. She has stopped the oral contraceptive pill some 6
months earlier. There is no past medical history of note. Examination reveals a BP of 105/72 mmHg, pulse is 62/min and regular. Her BMI is 19. She has normal breast
development and secondary sexual hair. Investigations;
Hb 11.9 g/dl
WCC 5.9 x10
9
/l
PLT 201 x10
9
/l
Na
+
137 mmol/l
K
+
4.3 mmol/l
Glucose 4.5 mmol/l
TSH 0.7 U/l
FSH 60 U/l
Oestradiol 25 pmol/l
Which of the following is the most likely diagnosis?
A Atypical congenital adrenal hyperplasia
B Polycystic ovarian syndrome
C Primary amenorrhoea due to Turners syndrome
D Secondary amenorrhoea due to low body weight CORRECT ANSWER
E Testicular feminisation
The answer is Secondary amenorrhoea due to low body weight - This woman has started her menstrual cycle, so this is by definition secondary amenorrhoea.
As such testicular feminisation cannot be the underlying cause. There are no symptoms of acne or hirsuitism and the patient has a low normal BMI, as such
Polycystic Ovarian Syndrome is extremely unlikely. Congenital Adrenal Hyperplasia is also more likely to be associated with symptoms of androgenisation.
This leaves secondary amenorrhoea, which may be related to this patients low weight, as the most appropriate diagnosis.
Specialty: MRCP 1 May 2014
Question: 99
A 54-year-old alcoholic comes to the Neurology Clinic for review. He apparently has drunk up to 500ml of whisky per day for many years, although his wife claims that
he has not drunk alcohol for the past 2 months after being weaned off it by his GP using a reducing course of Chlordiazepoxide. His wife is very worried as he appears to
be becoming increasingly confused. On examination his BP is 122/72 mmHg, pulse is 85/min (atrial fibrillation). He has signs of chronic liver disease although the
abdomen is soft and non-tender, his BMI is 23.Investigations;
Hb 12.7 g/dl
WCC 5.9 x10
9
/l
PLT 178 x10
9
/l
Na
+
137 mmol/l
K
+
4.0 mmol/l
Creatinine 88 micromol/l
ALT 85 U/l
ALP 165 U/l
Bilirubin 12 micromol/l
Which of the following would be most suggestive of an underlying diagnosis of Korsakoffs psychosis?
A Auditory hallucinations
B Inability to form new memories CORRECT ANSWER
C Loss of memory of distant events
D Tactile hallucinations
E Visual hallucinations
The answer is Inability to form new memories - Whilst some loss of memory of distant events is preserved, inability to form any new memories after the onset
of Korsakoffs psychosis is characteristic of the disorder. Other features include confabulation (falsification of memory in clear consciousness), telescoping of
time, (believing events which occurred years ago occurred much more recently), and inability to learn new tasks. Hallucinations are a feature of alcohol
withdrawal.

Das könnte Ihnen auch gefallen